Pathopharm II Exam 1

Ace your homework & exams now with Quizwiz!

The signs and symptoms of abrupt cessation of pharmacologic glucocorticoids closely resemble those of:

Addison disease. Feedback: Although the etiology differs, the adrenal cortical insufficiency resulting from the abrupt cessation of glucocorticoids is nearly identical to Addison disease in terms of physiologic effects.

Which of the following clinical manifestations would support the medical diagnosis of Cushing syndrome? Select all that apply.

Excessive facial hair growth, Blood glucose level in 200 mg/dL range, and "Buffalo hump" on back. Feedback: The major manifestations of Cushing syndrome represent an exaggeration of the many actions of cortisol. There is muscle weakness, and the extremities are thin. Derangements in glucose metabolism are found in approximately 75% of clients, with clinically overt diabetes mellitus occurring in approximately 20% of clients. The glucocorticoids possess mineralocorticoid properties; this causes fluid retention and hypertension resulting from sodium retention, water retention, and hypervolemia. An increase in androgen levels causes hirsutism. Altered fat metabolism causes a peculiar deposition of fat characterized by a protruding abdomen; subclavicular fat pads or "buffalo hump" on the back; and a round, plethoric "moon face."

The nurse is discussing upper and lower UTIs. Which of these conditions can be treated at home? (Select all that apply.)

a. Acute cystitis b. Acute urethritis c. Recurrent UTI ANS: A, B, C Acute cystitis, urethritis, and recurrent UTIs can be treated on an outpatient basis. Severe pyelonephritis and acute bacterial prostatitis require hospitalization and IV antibiotics.

As part of maintaining homeostasis, hormones secreted by endocrine cells are inactivated continuously to:

Prevent accumulation. Feedback: Continuous inactivation of secreted hormones is necessary to prevent accumulation that could disrupt the feedback mechanism. Increased secretion stimulates production of more receptor sites. Metabolic waste absorption is not a function of the endocrine system.

Nitric oxide, a vasodilator produced by the vascular endothelium, is important in renal control by: Select all that apply.

Preventing excessive vasoconstriction. Allowing normal excretion of sodium and water. Feedback: Nitric oxide, a vasodilator produced by the vascular endothelium, appears to be important in preventing excessive vasoconstriction of renal blood vessels and allowing normal excretion of sodium and water. Prostaglandins do not appear to play a major role in regulating renal blood flow. Some medications like aspirin and NSAIDs inhibit prostaglandin synthesis.

While explaining the tubular role in reabsorption, the nurse will stress that a diet high in sodium intake will result in sodium reabsorption in which of the following renal locations?

Proximal convoluted tubule. Feedback: Approximately 65% of all absorptive and secretory processes that occur in the tubular system take place in the proximal tubule. Electrolytes, such as Na+, K+, Cl-, and bicarbonate (HCO3), are 65% to 80% reabsorbed in this location.

A client had excessive blood loss and prolonged hypotension during surgery. His postoperative urine output is sharply decreased, and his blood urea nitrogen (BUN) is elevated. The most likely cause for the change is acute:

Tubular necrosis. Feedback: Ischemic acute tubular necrosis (ATN) occurs most frequently in persons who have major surgery with prolonged renal hypoperfusion—this directly damages the tubular epithelial cells with acute suppression of renal function. Nephrotoxic ATN is caused by toxic agents or drugs. Prerenal vasoconstriction is associated with acute-onset loss of renal output. Bladder (postrenal) obstruction would not affect the BUN, since it rarely causes renal failure.

A client with many nonspecific complaints has been ordered a positron emission tomography (PET) scanning for evaluation of:

Tumors located on the endocrine glands. Feedback: Positron emission tomography (PET) scanning is being used more widely for evaluation of endocrine tumors. PET scans do not calculate the pancreas response to insulin. A DEXA is used for diagnosis and monitoring of osteoporosis (bone density). Isotopic imaging includes radioactive scanning of the thyroid (using radioiodine) and parathyroids.

Which of the following signs and symptoms in a 2-year-old child should prompt assessment for a urinary tract infection?

Unexplained fever and anorexia. Feedback: Although all of the cited symptoms warrant further assessment and follow-up, the presence of fever and anorexia is typical of UTIs in toddlers.

Although the distal portion of the urethra often contains pathogens, the urine formed in the kidney and found in the bladder is sterile because of the:

Washout phenomenon. Feedback: Although the distal portion of the urethra often contains pathogens, the urine formed in the kidneys and found in the bladder normally is sterile or free of bacteria. This is because of the washout phenomenon, in which urine from the bladder normally washes bacteria out of the urethra during voiding. Because most urinary tract infection (UTI) bacteria ascend through the urethra, glomerular filtration cannot remove it. Pathogens are less likely to survive in acidic conditions; struvite calculi form secondary to bacterial infections causing alkaline urine. The core body temperature is not high enough to destroy bacterial microorganisms in the urine.

The nurse is caring for a patient receiving desmopressin [Stimate]. The nurse is performing a physical assessment and notes that the patient is drowsy, listless, and complains of a recent onset of headache. Which finding would be most consistent with these symptoms?

Water intoxication Water intoxication is manifested by drowsiness, listlessness, and headache. The patient's signs and symptoms are not consistent with hypernatremia or hyperglycemia, and there is no indication of hypertension.

A nurse teaches a nursing student about the differences between desmopressin (DDAVP) and vasopressin [Pitressin). Which statement by the student indicates a need for further teaching?

"Desmopressin has a shorter duration of action than vasopressin." Desmopressin has a long duration of action, which is the reason it is preferred for the treatment of diabetes insipidus. It can be administered intranasally, so it is easier to administer than vasopressin. Vasopressin has hemodynamic effects that can be beneficial during cardiac resuscitation but that also can cause serious adverse cardiovascular effects.

An adult client has been diagnosed with polycystic kidney disease. Which of the client's following statements demonstrates an accurate understanding of this diagnosis?

"I suppose I should be tested to see if my children might inherit this." Feedback: Autosomal dominant polycystic kidney disease is the most common of all inherited kidney diseases. The disorder is characterized by multiple expanding cysts of both kidneys that ultimately destroy the surrounding kidney structures and cause renal failure. The etiology of polycystic kidney disease (PKD) is not infectious, and it is not caused by nephrotoxic drugs or uncontrolled hypertension.

A male patient is being treated for benign prostatic hyperplasia (BPH) by another provider but cannot remember which drug he is taking. He comes to the clinic seeking treatment for erectile dysfunction and receives a prescription for sildenafil [Viagra). What will the nurse teach this patient?

"You should not take sildenafil if you are taking silodosin [Rapaflo]." Sildenafil should not be taken with alphaj-adrenergic antagonists, such as silodosin, because of the risk of severe decreases in blood pressure. Finasteride is not an alphaj-adrenergic antagonist and may be taken with sildenafil. Not all medications for BPH are safe to take with sildenafil. Invasive treatments for BPH are used when symptoms are severe.

A 30-year-old male patient reports having two to four urinary tract infections a year. What will the nurse expects to teach this patient?

"You will need to take a low dose of medication for 6 months to prevent infections." This patient has reinfection of his urinary tract at a rate of more than three per year, which is an indication for long-term prophylaxis. Voiding after intercourse is a good teaching point for sexually active women to prevent urinary infections, but it is not a sufficient preventive measure for recurrent infections in men. Short-course therapy may be used for each occurrence of infection if the reinfection rate is less than three per year. Long-term treatment for individual infections is recommended if relapse occurs or if infections do not clear with shorter-term therapy.

The parent of a 5-year-old child who has had four urinary tract infections in the past year asks the nurse why the provider doesn't just order an antibiotic for the child's current symptoms of low-grade fever, flank pain, and dysuria since these are similar symptoms as before. Which is the most important reason given by the nurse?

"Your child may need tests to assess for urinary tract abnormalities." Children with recurrent urinary tract infections should be assessed for underlying urinary tract abnormalities to help determine a possible cause for recurrence. This child has mild fever and therefore may not require hospitalization. Urine cultures are important when treating patients with recurrent UTI prophylactically, but this is not the most important consideration. Urinary tract antiseptics are used to treat uncomplicated lower urinary tract infections.

A bladder cancer client asks the nurse, "What did the doctor mean by intravascular chemotherapy? Am I going to lose all my hair and have to do for treatments over months and months?" The best response would be:

"This is when they put the chemotherapy directly into the bladder to kill any cancer cells." Feedback: Surgical treatment of superficial bladder cancer is often followed by intravascular chemotherapy or immunotherapy, a procedure in which the therapeutic agent is directly instilled into the bladder. None of the other responses describe this procedure. The chemotherapy drug is not injected through the abdomen into the bladder. BCG is instilled into the bladder to elicit an inflammatory response that can kill the tumor. A CyberKnife is used with the brain, not the bladder.

A patient has a free T4 level of 0.6 ng/dL and a free T3 of 220 pg/dL. The patient asks the nurse what these laboratory values mean. How will the nurse respond?

"We will need to obtain a TSH level to better evaluate your diagnosis." A free T4 level of less than 0.9 ng/dL and a free T3 of less than 230 pg/dL are consistent with hypothyroidism, but measurement of the thyroid-stimulating hormone (TSH) level is necessary to distinguish primary hypothyroidism from secondary hypothyroidism. Total T3 and T4 levels are not as helpful as free T3 and T4 levels. These laboratory values indicate hypothyroidism, not hyperthyroid conditions such as Graves' disease.

A patient who has been newly diagnosed with adrenal hormone deficiency will begin taking hydrocortisone. The nurse provides teaching for this patient. Which statement by the patient indicates understanding of the teaching?

"When I am sick, I should take 3 times the normal dose for 3 days in a row." Patients who take hydrocortisone as replacement need to be taught to increase their daily dose during times of stress, because the drug causes adrenal suppression, and the adrenals will not be able to release cortisone as usual during stress. A general rule of thumb is the "3 by 3 rule": take 3 times the usual dose for 3 days when sick. The daily dose is usually given once in the morning; if late-day fatigue occurs, patients may split the dose and take two-thirds in the morning and one third in the late afternoon or evening. Treatment is lifetime. Side effects are uncommon when hydrocortisone is given in therapeutic doses.

A young, nonpregnant female patient with a history of a previous urinary tract infection is experiencing dysuria, urinary urgency and frequency, and suprapubic pain of 3 days' duration. She is afebrile. A urine culture is positive for more than 100,000/mL of urine. The nurse caring for this patient knows which treatment is most effective?

A 3-day course of trimethoprim/sulfamethoxazole [Bactrim] Short-course therapy is recommended for uncomplicated, community-acquired lower urinary tract infections. The short course is more effective than a single dose, and compared with longer course therapies, it is less costly, has fewer side effects, and is more likely to foster compliance. Amoxicillin with clavulanic acid is a second-line drug used for pyelonephritis. Fosfomycin is a second-line drug and can be useful in patients with drug allergies.

Which of the following clients is likely at the greatest risk of developing a urinary tract infection?

A 79-year-old client with an indwelling catheter for urinary incontinence. Feedback: There is an increased risk for UTIs in persons with urinary obstruction and reflux, in people with neurogenic disorders that impair bladder emptying, in women who are sexually active, in postmenopausal women, in men with diseases of the prostate, and in elderly persons. Instrumentations and urinary catheterization are the most predisposing factors for nosocomial, or hospital-acquired UTIs. Frequency and incontinence may be signs and symptoms of UTIs, but they are not causative of the infections.

When trying to explain hypothyroidism to a newly diagnosed client, the nurse stresses the fact that the thyroid hormone is transported in blood by specific:

A) Proteins. Feedback: Some hormones, such as steroids and thyroid hormone, are bound to protein carriers for transportation to the target cell destination. The extent of carrier binding influences the rate at which hormones leave the blood and enter the cells. Cholesterol is a precursor for steroid hormones. Prohormones have an extra amino acid and are converted to hormones in the Golgi complex.

Which of the following physiologic processes best exemplifies a positive feedback mechanism?

A) The increase in prolactin secretion that occurs with more frequent breast-feeding. Feedback: A positive feedback mechanism occurs when one hormonal or physiologic factor stimulates further hormonal release, creating a cascade that will increase until corrected. Correction of alterations in homeostasis is normally achieved using negative feedback mechanisms, such as those accomplished by insulin, parathyroid hormone, and ADH.

A diabetic client with a history of hypertension may receive a prescription for which medication to provide a renal protective effect by reducing intraglomerular pressure? Select all that apply.

ACE inhibitors and Angiotensin receptor blockers. Feedback: The ACE inhibitors and ARBs reduce the effects of angiotensin II on renal blood flow. They also reduce intraglomerular pressure and may have a renal protective effect in persons with hypertension or type 2 diabetes. However, when combined with diuretics, they may cause prerenal injury in persons with decreased blood flow due to large-vessel or small-vessel kidney disease. Calcium channel blockers are vasodilators.

An older adult patient who lives alone and is somewhat forgetful has an overactive bladder (OAB) and reports occasional constipation. The patient has tried behavioral therapy to treat the OAB without adequate results. Which treatment will the nurse anticipate for this patient?

Oxybutynin [Oxytrol] transdermal patch. The transdermal patch is applied weekly and may be the best option for a patient who is more likely to forget to take a daily medication. The transdermal preparation has fewer side effects than the systemic dose, so it is less likely to increase this patient's constipation. The syrup has a high incidence of dry mouth and other anticholinergic side effects. The extended-release tablets must be given daily, and this patient may not remember to take them. PTNS is used after behavioral and drug therapies have failed.

A newly diagnosed paraplegic client who suffered an automobile accident appears to have control of bladder emptying. The health care provider explains this process to the client/family stating, "This function is allowing the motor component of the neural reflex to assist with bladder emptying and is primarily controlled by the:

Parasympathetic division of the ANS. Feedback: The motor component of the neural reflex to assist with bladder emptying is primarily controlled by the parasympathetic division of the ANS, and the relaxation and storage functions of the bladder are controlled by the sympathetic division. The somatic nervous system innervates the skeletal muscles of the external sphincter and the pelvic floor muscles that together control the outflow of urine. The afferent input from the bladder and urethra is carried to the CNS by fibers that travel with PS, somatic, and sympathetic (hypogastric) nerves.

The nurse understands that muscarinic antagonists are often referred by other names. Which names should the nurse recognize for this category of drug?

Parasympatholytic drugs. Anticholinergic drugs. Antimuscarinic drugs. Muscarinic blockers. The nurse should recognize that following as being synonymous with muscarinic antagonists: parasympatholytic, anticholinergic, antimuscarinic, and Muscarinic blockers.

When explaining to a CKD client how urea is absorbed, which of the following transport mechanisms will be mentioned?

Passive transport. Feedback: The mechanisms of transport across the tubular cell membrane are similar to those of other cell membranes in the body and include active and passive transport mechanisms. Water and urea (a by-product of protein metabolism) are passively absorbed along concentration gradients. Sodium (Na+), other electrolytes, as well as urate, glucose, and amino acids, are reabsorbed using primary or secondary active transport mechanisms to move across the tubular membrane. The bulk of energy used by the kidney is for active sodium transport mechanisms that facilitate sodium reabsorption and cotransport of other electrolytes and substances such as glucose and amino acids.

Following an episode of strep throat, the school nurse notices the fourth grade child has not recovered from this illness a week later. Upon further investigation, the nurse notices that the child has developed water retention. Which of the following assessments support this conclusion? Select all that apply.

Periorbital edema and a BP 100/70. Feedback: Generalized edema, a hallmark of nephrosis, results from salt and water retention and a decrease in plasma colloid osmotic pressure due to loss of albumin in the urine. Glomerulonephritis is characterized by sodium and water retention that causes edema, particularly of the face and hands. Fluid retention usually results in an elevated BP, not a normal one. Vomiting and dizziness are not associated with this diagnosis.

While assessing a peritoneal dialysis client in his or her home, the nurse notes that the fluid draining from the abdomen is cloudy, is white in color, and contains a strong odor. The nurse suspects this client has developed a serious complication known as:

Peritonitis. Feedback: Potential problems with peritoneal dialysis include infection, catheter malfunction, dehydration, hyperglycemia, and hernia. Bowel perforation can occur, but the fluid would be stool colored. The client may develop hyperglycemia; however, this will not cause the fluid to be cloudy. If bladder erosion had occurred, the fluid would look like urine and not be cloudy and white.

A patient has developed muscarinic antagonist toxicity from ingestion of an unknown chemical. The nurse should prepare to administer which medication?

Physostigmine [Antilirium] Physostigmine is indicated for muscarinic antagonist toxicity. Atropine is a drying agent and would only complicate the drying action that arises from the muscarinic antagonist. An acetylcholinesterase activator would only contribute to dryness that arises from the muscarinic antagonist. Ephedrine is not indicated for muscarinic antagonist toxicity.

Which of the following diagnostic and assessment results support the diagnosis of chronic pyelonephritis? Select all that apply.

Polyuria (excess urine output) Nocturia (voiding at night). Feedback: The symptoms of chronic pyelonephritis often include a history of recurrent episodes of UTI or acute pyelonephritis. Loss of tubular function and the ability to concentrate urine give rise to polyuria and nocturia, and mild proteinuria is common. Severe hypertension often is a contributing factor in the progress of the disease. A BP of 140/92 is not considered "severe" hypertension. Flank and upper outer quadrant pain is usually associated with kidney stones.

When acute tubular necrosis (ATN) is suspected, the nurse will likely see which of the following laboratory findings on the urinalysis report? Select all that apply.

Protein, Red blood cells, and Cast cells. Feedback: Nephron damage allows the larger protein cells to pass through the membrane and into the urine (normally, urine has very few proteins present). Further diagnostic information that can be obtained from the urinalysis includes hemoglobinuria (blood in the urine) and casts or crystals in the urine. Glucosuria in the urine is an indirect indication of extreme hyperglycemia, often unrelated to renal disease. Urine sodium concentration is maintained with prerenal azotemia; urine sodium decreases with renal tubule damage. Urine calcium is not diagnostic for ATN

Which of the following assessment findings would lead the nurse to suspect the client has nephrotic syndrome?

Proteinuria and generalized edema. Feedback: The nephrotic syndrome is characterized by massive proteinuria and lipiduria, along with an associated hypoalbuminemia, generalized edema, and hyperlipidemia.

An elderly female client has been hospitalized for the treatment of acute pyelonephritis. Which of the following characteristics of the client is most likely implicated in the etiology of her current health problem? The client:

Recently had a urinary tract infection. Feedback: There are two routes by which bacteria can gain access to the kidney: ascending infection from the lower urinary tract and through the bloodstream. Ascending infection from the lower urinary tract is the most important and common route by which bacteria reach the kidney, resulting in acute pyelonephritis. Diabetes, hypertension controlled by a diuretic and an ACE inhibitor, and peripheral vascular disease are not associated with acute pyelonephritis.

Which one of the following blood tests reflects the glomerular filtration rate (GFR) and is used to estimate renal function?

Serum creatinine. Feedback: Serum creatinine level is used to estimate functional capacity of the kidneys. Increased creatinine level indicates decreased GFR and renal function. Blood urea nitrogen (BUN) levels are influenced by hydration status, protein intake, and bleeding, in addition to renal function. Serum ammonia is a metabolic by-product of urea and can be influenced by multiple factors unrelated to kidney function.

A client who has been diagnosed with Addison disease will likely experience which of the following lab results related to the absence of aldosterone?

Serum potassium levels elevated. Feedback: Aldosterone exerts a strong influence on potassium secretion in the distal and collecting tubules. In the absence of aldosterone, as occurs in Addison disease, potassium secretion is markedly decreased, causing blood levels to increase. In the presence of aldosterone, almost all the sodium in the distal tubular fluid is reabsorbed, and the urine essentially becomes sodium free. In the absence of aldosterone, virtually no sodium is reabsorbed from the distal tubule, and excessive amounts of sodium are lost in the urine.

A chronic kidney disease (CKD) client asks the nurse, "Why do I itch all the time?" The nurse bases her response on which of the following integumentary physiologic factors that causes pruritis? Select all that apply.

Decrease in perspiration and Elevated serum phosphate levels. Feedback: Dry, itchy skin is a common consequence of CKD. Pruritus is common; it results from the high serum phosphate levels and the development of phosphate crystals that occur with hyperparathyroidism. Harsh soap (may dry the skin), limited Na+ intake, and enlarged sweat glands are not noted to accompany or result in pruritus.

If a CKD client is developing uremic encephalopathy, the earliest manifestations may include: Select all that apply.

Decreased alertness and Diminished awareness. Feedback: Reductions in alertness and awareness are the earliest and most significant indications of uremic encephalopathy. Late in the disease process, the client may develop delirium, coma, and seizures.

Client and family education regarding peritoneal dialysis should include assessing the client for:

Dehydration that may appear as dry mucous membranes or poor skin turgor. Feedback: Potential problems with peritoneal dialysis include infection, catheter malfunction, dehydration caused by excessive fluid removal, hyperglycemia, and hernia. The most serious complication is infection, which can occur at the catheter exit site, in the subcutaneous tunnel, or in the peritoneal cavity. In peritoneal dialysis, a sterile dialyzing solution is instilled through a catheter over a period of approximately 10 minutes. Then the solution is allowed to remain in the peritoneal cavity for a prescribed amount of time. Shunts, fistulas, and artificial dialyzers are associated with hemodialysis, which is usually performed three times weekly.

A client's most recent blood work reveals a blood urea nitrogen (BUN) level of 36 mg/dL (normal range 8 to 25 mg/dL). Which of the following factors may have contributed to this finding?

Dehydration. Feedback: During periods of dehydration, the blood volume and glomerular filtration rate (GFR) drop, and BUN levels increase. Increased salt intake, parasympathetic stimulation, and the action of ADH do not normally result in an increase in BUN.

The nurse should anticipate that a client who collapsed while running his or her first marathon and has a urine specific gravity of 1.035 is experiencing:

Dehydration. Feedback: The usual range of specific gravity is 1.010 to 1.025 with normal fluid intake. Healthy kidneys can produce concentrated urine with specific gravity of 1.030 to 1.040 during periods of dehydration and dilute urine with a specific gravity that approaches 1.000 during periods of too much fluid intake. Frostbite, sunstroke, and exhaustion do not change urine specific gravity if fluid volume is normal.

Manifestations of childhood renal disease are varied and may differ from adult-onset renal failure. A school-aged child with chronic kidney disease may exhibit:

Developmental delays such as uncoordinated gait and minimal fine motor skills. Feedback: Childhood chronic kidney disease is manifested by growth and developmental delays and late onset sexual maturity as a result of the uremic effects on endocrine function, bone abnormalities, and development of psychosocial problems. Renal failure is unrelated to the ability of children to have control of urine or bowel elimination. Intelligence is not affected by renal failure, although renal encephalopathy may affect behavior.

A patient is hospitalized with head trauma after a motor vehicle accident. The nurse caring for the patient notes a marked increase in the output of pale, dilute urine. The nurse suspects which condition?

Diabetes insipidus Deficiency of antidiuretic hormone (ADH) produces hypothalamic diabetes insipidus, in which large volumes of dilute urine are produced. Head trauma can cause the hypothalamus to stop producing ADH. Diabetes mellitus is an endocrine disorder of the pancreas that causes the production of large volumes of non dilute urine. SIADH is a condition in which too much ADH is produced, causing oliguria. Water intoxication occurs with SIADH.

urinary tract infection (UTI). Which of the following factors most likely predisposed this client to the development of a UTI?

Dilation of the upper urinary structures. Feedback: Normal changes in the functioning of the urinary tract that occur during pregnancy predispose to UTIs. These changes involve the collecting system of the kidneys and include dilation of the renal calyces, pelves, and ureters that begins during the first trimester and becomes most pronounced during the third trimester. Bladder hypertrophy, spastic peristalsis, and increased urine pH are not phenomena that are common accompaniments to pregnancy.

A nurse is preparing to administer a dose of growth hormone and reconstitutes the medication. After adding the diluent, the nurse notices that the preparation is cloudy. What will the nurse do?

Discard the drug and prepare another dose. The medication should not be injected if the preparation is cloudy or contains particulate matter. The drug should be discarded and another dose drawn up and administered. There is no need to notify the prescriber. The preparation should never be shaken.

A client is beginning to recover from acute tubular necrosis. The nurse would likely be assessing which of the following manifestations of the recovery phase of ATN?

Diuresis. Feedback: The recovery phase is first noticed as increased/excessive output (diuresis) of dilute urine and a fall in serum creatinine, indicating that the nephrons have recovered to the point at which urine excretion is possible. Potassium will remain elevated or continue to rise, since the diuresis occurs before renal function fully returns to normal. Edema/fluid retention is characteristic of the maintenance phase. Proteinuria is characteristic of glomerular disease and/or chronic kidney disease.

A patient with acromegaly asks the nurse about treatments for this condition. What will the nurse tell the patient?

Drugs are generally used after surgical and radiation therapies have been tried. A Drugs for acromegaly are generally reserved for patients who do not respond to other therapies or for whom these therapies are not viable options. Drug therapy requires daily subQ injections and is expensive. Drug therapy is indefinite, not short term.

Before giving methenamine [Hiprex] to a patient, it is important for the nurse to review the patient's history for evidence of which problem?

Elevated blood urea nitrogen and creatinine Methenamine should not be given to patients with renal impairment, because crystalluria can occur. There is no cross-reactivity between methenamine and antibiotic agents. Methenamine is safe for use during pregnancy. There is no organism drug resistance to methenamine. A patient who is taking nitrofurantoin calls the nurse to report several side effects. Which side effect of this drug causes the most concern and would require discontinuation of the medication? a. Anorexia, nausea, and vomiting b. Brown-colored urine c. Drowsiness d. Tingling of the fingers ANS: D Tingling of the fingers can indicate peripheral neuropathy, which can be an irreversible side effect of nitrofurantoin. The other side effects are not serious and can be

A young child has been diagnosed with Wilms tumor after his mother discovered an unusual mass, prompting a diagnostic workup. Which of the following characteristics is typical of Wilms tumor? The tumor is usually:

Encapsulated. Feedback: Wilms tumor usually is a solitary mass that occurs in any part of the kidney. It usually is sharply demarcated and variably encapsulated. It is not a self-limiting health problem, and chemotherapy, radiotherapy, and/or surgery may be utilized. Symptoms include hypertension, abdominal pain, and vomiting.

If a client with a kidney stone has the "classic" ureteral colic, the client will describe his pain as: Select all that apply.

Acute, intermittent, Excruciating, and In the flank and upper outer quadrant of the abdomen. Feedback: The symptoms of renal colic are caused by stones 1 to 5 mm in diameter that can move into the ureter and obstruct flow. Classic ureteral colic is manifested by acute, intermittent, and excruciating pain in the flank and upper outer quadrant of the abdomen on the affected side. The pain may radiate to the lower abdominal quadrant, bladder area, perineum, or scrotum in the man. The pain is usually not described as diffuse and over the entire low back and legs.

While studying for a renal test in pathophysiology class, a student helps a peer by reviewing facts about the cells of the proximal tubule. Which of the following functions should they include in this discussion? Select all that apply.

Aids in reabsorption. Rich in mitochondria. Supports active transport processes. Feedback: The cells of the proximal tubule have a fine, villous structure that increases the surface area for reabsorption; they also are rich in mitochondria, which support active transport processes. The epithelial layer thins in segments of the loop of Henle and has few mitochondria, indicating minimal metabolic activity and resorptive function.

When discussing luteinizing hormone and follicle-stimulating hormone with students, the instructor will emphasize that these hormones are under the control of:

Anterior pituitary gland. Feedback: The pituitary gland has been called the master gland because its hormones control the functions of many target glands and cells. The anterior pituitary gland or adenohypophysis contains five cell types: (1) thyrotropes, which produce thyrotropin, also called TSH; (2) corticotropes, which produce corticotropin, also called ACTH; (3) gonadotrophs, which produce the gonadotropins, LH, and FSH; (4) somatotrophs, which produce GH; and (5) lactotrophs, which produce prolactin.

Glomerulonephritis is usually caused by:

Antigen-antibody complexes. Feedback: Two types of immune mechanisms have been implicated in the development of glomerular disease: injury resulting from antibodies reacting with fixed glomerular antigens and injury resulting from circulating antigen-antibody complexes that become trapped in the glomerular membrane. Reflux, which is the most common cause of chronic pyelonephritis, results from superimposition of infection on congenital vesicoureteral reflux or intrarenal reflux. Urinary catheters provide a means for microorganisms to ascend into the urinary tract to cause bladder infections or pyelonephritis.

A client with a diagnosis of chronic kidney disease (CKD) may require the administration of which of the following drugs to treat coexisting conditions that carry a high mortality?

Antihypertensive medications. Feedback: Hypertension is a common result of CKD, and the mechanisms that produce hypertension in CKD include increased vascular volume, elevation of peripheral vascular resistance, decreased levels of renal vasodilator prostaglandins, and increased activity of the renin-angiotensin-aldosterone system. NSAIDs, opioids, and antiarrhythmics are not as frequently indicated for the treatment of CKD.

A patient with arthritis is admitted to the hospital. The patient's serum glucose level is 350 gm/dL, and the blood pressure is 182/98 mm Hg. The nurse notes that the patient's face appears rounded and puffy. The patient complains of feeling weak. What will the nurse do?

Ask which drugs the patient takes for arthritis. Many patients with arthritis are treated with glucocorticoids. Because the doses necessary to suppress inflammation are larger than the physiologic doses used to treat adrenal insufficiency, patients can develop signs of cortisol excess with cushingoid symptoms. This patient has an elevated glucose level, hypertension, and the characteristic moon facies and muscle weakness of Cushing's syndrome; therefore, the nurse would be correct to ask about the medications the patient takes for arthritis. A pituitary carcinoma could be the cause but is less likely. Ketoconazole is used after chemotherapy and radiation therapy in the treatment of a pituitary adenoma. This patient does not show signs of Addison's disease.

A client fell off a ladder and sustained a spinal cord injury that has resulted in bladder dysfunction. During the period immediately after the spinal injury, spinal shock develops and the bladder displays what type of function?

Atonic. Feedback: Immediately following spinal cord injury, a state of spinal shock develops in which all reflexes, including the micturition reflex, are depressed. During this stage, the bladder becomes atonic and cannot contract. Spasmodic, hyperactive, and uninhibited bladder function occurs after the acute stage of spinal cord injury and during the recovery stage.

The release of insulin from the pancreatic beta cells can inhibit its further release from the same cells. This is an example of which type of hormone action?

Autocrine. Feedback: Hormones can exert autocrine action on the cells from which they were produced. Retinoids are compounds with hormone-like actions. Juxtracrine action involves a chemical messenger embedded in a plasma membrane that interacts with a specific receptor on a juxtaposed cell. Arachidonic acid is a precursor for eicosanoid compounds (similar to retinoids).

A patient who has erectile dysfunction asks about medications to treat this disorder, but tells the nurse he does not want to have to plan sexual activity several hours in advance. Which medication will the nurse expect the provider to order for this patient?

Avanafil [Stendra] Avanafil is used to treat ED and, unlike the other agents, has a shorter onset of action, with effects occurring in 15 minutes after taking the drug and may be taken 30 minutes prior to intercourse. The other agents must be taken 2 hours prior to intercourse.

Protein and blood cell leakage into the filtrate that occurs in many forms of glomerular disease is a result of changes in the structure and function of the glomerular:

Basement membrane Feedback: The basement membrane is a meshwork of collagen fibers, with slit pores between the fibers creating size-dependent permeability that (normally) does not allow large molecules, such as protein and blood cells through. The renal corpuscle contains the Bowman's capsule and capillaries surrounding it. Peritubular capillary network is a low-pressure resorptive system that permits rapid fluid/solute transfer to/from the tubules.

Although urinary obstruction and urinary incontinence have almost opposite effects on urination, they can both result from:

Bladder structure changes. Feedback: Disorders of lower urinary tract structure and function include urinary obstruction with retention or stasis of urine and urinary incontinence with involuntary loss of urine. Both types of disorders can have their origin in the structures of the lower urinary tract or in the neural mechanisms that control their function. Urinary incontinence can result from loss of bladder distensibility. Chronic outlet obstruction can cause bladder wall hypertrophy. Incontinence can result from reflex spasms, leading to segmental reflex bladder control instead of micturition center control.

blood pressure. While educating the client about the actions of the medication, the nurse will mention which of the following actions? Select all that apply.

Blocks Na+ reabsorption in distal tubules. Increases active reabsorption of Ca++ into the blood. Feedback: The thiazide diuretics, which are widely used to treat disorders such as hypertension, exert their action by blocking sodium reabsorption in this segment of the renal tubules, while enhancing the active reabsorption of calcium into the blood via the exchange transport mechanism. For this reason, thiazide diuretics have proved useful in reducing the incidence of calcium kidney stones in persons with hypercalciuria. ATPase pump maintains a low sodium concentration inside the cell by moving sodium down its concentration into the cell through special sodium channels. The pump also establishes a high concentration of potassium within the cell, causing it to diffuse down its concentration gradient across the luminal membrane into the tubular fluid.

Regardless of the cause, chronic kidney disease results in progressive permanent loss of nephrons and glomerular filtration, and renal:

Endocrine functions. Feedback: Chronic kidney disease results in loss of nephrons, tubule, and endocrine functions such as erythropoietin production. Systemic and renal hypertension is commonly an early manifestation of chronic kidney disease, caused by resistance to blood flow through the constricted renal vessels. Tubule hypertrophy is a compensatory response for those destroyed—when the few remaining nephrons are destroyed, renal failure is apparent. Phosphate accumulates in the blood; since it is inversely related to calcium, the levels of which remain chronically low.

Which of the following hormones are synthesized by non-vesicle-mediated pathways? Select all that apply.

Estrogen and Aldosterone. Feedback: Glucocorticoids, androgens, estrogens, and mineralocorticoids (aldosterone is an example of a mineralocorticoid) are all hormones synthesized by non-vesicle-mediated pathways. Epinephrine and insulin are synthesized by vesicle-mediated pathways.

Which of the following pathophysiologic phenomena may result in a diagnosis of Cushing disease?

Excess ACTH production by a pituitary tumor. Feedback: Three important forms of Cushing syndrome result from excess glucocorticoid production by the body. One is a pituitary form, which results from excessive production of ACTH by a tumor of the pituitary gland. Hypopituitarism and destruction of the adrenal cortex are associated with Addison disease. Disruption of the HPA system is not implicated in the etiology of Cushing disease.

When sensors detect a change in a hormone level, the hormonal response is regulated by which of the following mechanisms that will return the level to within normal range,

Feedback. Feedback (negative and sometimes positive) mechanisms respond to levels that are too high or too low. The level of many of the hormones in the body is regulated by negative feedback mechanisms. The function of this type of system is similar to that of the thermostat in a heating system. For example, when the sensors detect a decrease in blood levels, they initiate changes that cause an increase in hormone production. Metabolic responses and increased/decreased production occur as a consequence of hormone level fluctuations, not as a regulating mechanism. Action potential is not a regulating mechanism for this function.

A client with significant burns on his lower body has developed sepsis on the 3rd day following his accident. Which of the following manifestations would the nurse anticipate for an ischemic acute tubular necrosis rather than prerenal failure? The client:

GFR does not increase after restoration of renal blood flow. Feedback: In contrast to prerenal failure, the glomerular filtration rate (GFR) does not improve with the restoration of renal blood flow in acute renal failure caused by ischemic acute tubular necrosis. Edema, oliguria, and hematuria are not diagnostic of acute tubular necrosis (ATN), and hemodialysis does not normally fail to achieve a reduction in blood urea nitrogen (BUN) and creatinine.

A nurse working in the emergency department is assigned to a child who is arriving by ambulance after being involved in a spill of organophosphate insecticides. What will the nurse expect to be the initial priority for treating this child?

Giving diazepam to control seizures Fluid retention. Diarrhea Acarbose and miglitol frequently cause flatulence, cramps, abdominal distention, borborygmus (rumbling bowel sounds), and diarrhea. Acarbose also can cause liver damage. Neither acarbose nor miglitol causes hypoglycemia, elevated iron levels, or fluid retention.

A nurse caring for a patient notes that the patient has a temperature of 104°F, and a heart rate of 110 beats/min. The patient's skin is warm and moist, and the patient complains that the room is too warm. The patient appears nervous and has protuberant eyes. The nurse will contact the provider to discuss:

Graves' disease. The signs and symptoms in this patient are consistent with hyperthyroidism, and because the patient's eyes are protuberant, they also are consistent with Graves' disease. Cretinism is hypothyroidism in children. Myxedema is severe hypothyroidism. Plummer's disease is a hyperthyroidism condition without exophthalmos.

A client has been recently undergone diagnostic testing for possible Berger disease. The nurse caring for this client would anticipate the primary clinical manifestations include which of the following? Select all that apply.

Gross hematuria, Recent upper respiratory infection, Fever, chills, and general body aches Feedback: Early in the disease, many people with the disorder have no obvious symptoms, and the disorder is discovered during screening or examination for another condition. In others, the disorder presents with gross hematuria that is preceded by upper respiratory tract infection, GI tract symptoms, or flulike illness. The hematuria lasts 2 to 6 days. Elevated ketones are usually associated with acidosis, fasting, high-protein diet, or diabetes to name a few.

Loss of pituitary function can result in deficiencies/loss of which of the following hormones' secretions? Select all that apply.

Growth hormone, Luteinizing hormone, Follicle stimulating hormone, and Prolactin Feedback: Anterior pituitary hormone loss is usually gradual, especially with progressive loss of pituitary reserve due to tumors or previous pituitary radiation therapy (which may take 10 to 20 years to produce hypopituitarism). The loss of pituitary function tends to follow a classic course beginning with the loss of GH, LH, and FSH secretion followed by deficiencies in TSH, then ACTH, and finally prolactin.

A nurse is obtaining a history from a patient who will receive tadalafil [Cialis] for erectile dysfunction. The patient reports that he expects to have sexual activity 3 or 4 times a week. What should the nurse tell the patient?

He should talk to his provider about daily dosing of tadalafil. Tadalafil is approved for daily dosing and can be given daily for men who anticipate sexual activity twice a week or more. Tadalafil has longer effects—up to 36 hours—than sildenafil, and adverse effects take longer to resolve. There is no increased risk of priapism associated with an increase in sexual activity. Because tadalafil has longer effects, dosing and sexual activity do not have to be closely timed.

While taking a history from an adult client newly diagnosed with renal cell cancer, the nurse can associate which of the following high-risk factors with the development of this cancer?

Heavy smoking. Feedback: Epidemiologic evidence suggests a correlation between heavy smoking and kidney cancer. Obesity also is a risk factor; particularly in women. The risk of renal cell carcinoma also is increased in persons with acquired cystic kidney disease associated with chronic renal insufficiency. Although the adrenal gland is adjacent to the kidney, primary adrenal tumors are unrelated to renal cell disease.

Which of the following client clinical manifestations most clearly suggests a need for diagnostic testing to rule out renal cell carcinoma?

Hematuria Feedback: Presenting features of renal cancer include hematuria, flank pain, and presence of a palpable flank mass. Gross or microscopic hematuria, which occurs in more than 50% of cases, is an important clinical clue. Urgency, oliguria, and cloudy urine are not as closely associated with renal carcinoma.

The nurse is providing care for a client who has a diagnosis of kidney failure. Which of the following laboratory findings is consistent with this client's diagnosis?

Hypocalcemia. Feedback: Diagnostic findings that are congruent with a diagnosis of kidney failure include hyperphosphatemia, hypocalcemia, a decrease in active vitamin D levels, and secondary hyperparathyroidism.

A nurse obtaining an admission history on an adult patient notes that the patient has a heart rate of 62 beats/min, a blood pressure of 105/62 mm Hg, and a temperature of 96.2°F. The patient appears pale and complains of always feeling cold and tired. The nurse will contact the provider to discuss tests for which condition?

Hypothyroidism. This patient is showing signs of hypothyroidism: a low heart rate, low temperature, pale skin, and feeling cold and tired. In adults, thyroid deficiency is called hypothyroidism. In children, thyroid deficiency is called cretinism. Graves' disease and Plummer's disease are conditions caused by thyroid excess.

A client has recently undergone successful extracorporeal shock wave lithotripsy (ESWL) for the treatment of renal calculi. Which of the following measures should the client integrate into his lifestyle to reduce the risk of recurrence?

Increased fluid intake and dietary changes. Feedback: Depending on the type of stone that was present, many clients benefit from increased fluid intake and changes in diet. Weight loss, blood sugar and pressure control, and exercise are not central preventative measures. It would likely be inappropriate to recommend the use of over-the-counter diuretics as a preventative measure.

The client has had prolonged urethral outlet obstruction. The nurse knows that physiologically, the client may likely develop small pockets of mucosal tissue, called cellulite, which can ultimately cause: Select all that apply.

Infections due to stasis, Backpressure on the ureters, and Development of hydroureters. Feedback: With continued outflow obstruction, this smooth surface is replaced with coarsely woven structures (i.e., hypertrophied smooth muscle fibers) called trabeculae. Small pockets of mucosal tissue, called cellulite, commonly develop between the trabecular ridges. These pockets form diverticula when they extend between the actual fibers of the bladder muscle. Because the diverticula have no muscle, they are unable to contract and expel their urine into the bladder, and secondary infections caused by stasis are common. Along with hypertrophy of the bladder wall, there is hypertrophy of the trigone area and the interureteric ridge, which is located between the two ureters. This causes backpressure on the ureters, the development of hydroureters and, eventually, kidney damage. Detrusor-sphincter dyssynergia is uncoordinated activity that causes overdistention. Sphincter dystonia is a cause of incontinence. Interstitial cystitis is a condition of increased sensitivity to bladder pressure, loss of bladder elasticity, and severe urgency unrelated to outlet obstruction.

A patient with Cushing's syndrome has undergone surgery and radiation treatment. The nurse will expect to teach the patient about which medication?

Ketoconazole (Nizoral] The role of drugs in the treatment of Cushing's syndrome is limited; drugs are used only as adjuncts to surgery and radiation therapy. The most effective agent is ketoconazole, which suppresses steroid synthesis. Cosyntropin is used to diagnose adrenal insufficiency. Dexamethasone is a glucocorticoid used for replacement therapy and to diagnose Cushing's syndrome. Fludrocortisone is used for chronic mineralocorticoid replacement.

A nursing student who has a history of brain tumors that resulted in partial removal of her pituitary gland years ago is asking her OB/GYN doctor about her ability to breast-feed her infant. This is based on which physiological function of the pituitary gland that facilitates breast milk production?

Lactotrophs Ans: D Feedback: The anterior pituitary gland or adenohypophysis contains five cell types: (1) thyrotropes, which produce thyrotropin, also called TSH; (2) corticotropes, which produce corticotropin, also called ACTH; (3) gonadotrophs, which produce the gonadotropins, LH, and FSH; (4) somatotrophs, which produce GH; and (5) lactotrophs, which produce prolactin that is involved with breast growth and milk production.

After reviewing the 24-hour intake and output of a hospital client, the nurse suspects that the client may be experiencing flaccid bladder dysfunction. Which of the following diagnostic methods is most likely to confirm or rule out whether the client is retaining urine?

Measurement of postvoid residual (PVR) by ultrasound. Feedback: Measurement of PVR can be achieved quickly, accurately, and painlessly by the use of ultrasonography. A PVR value of less than 50 mL is considered adequate bladder emptying, and more than 200 mL indicates inadequate bladder emptying. Urine tests and blood tests will not directly indicate whether a client is experiencing bladder fill with insufficiency in emptying.

One of the first signs that indicates an infant may have congenital hypothyroidism is:

C)Prolonged period of physiologic jaundice. Feedback: With congenital lack of the thyroid gland, the infant usually appears normal and functions normally at birth because of hormones supplied in utero by the mother. Prolongation of physiologic jaundice, caused by delayed maturation of the hepatic system for conjugating bilirubin, may be the first sign. There may be respiratory difficulties and a hoarse cry, feeding difficulties, and an enlarged abdomen. This condition will not interfere with meconium passage, elevated ICP resulting in full, tight fontanels, or having a palpable mass in the neck.

A male patient reports decreased libido and the nurse notes galactorrhea during a physical assessment. The nurse will report these findings to the provider and will anticipate an order for which medication?

Cabergoline The patient is showing signs of prolactin hypersecretion. Cabergoline is a dopamine agonist and is used to inhibit prolactin release. Conivaptan is used to treat hypernatremia. Dopamine is not given. Prolactin would make the condition worse.

A client is experiencing bladder hyperactivity. The nurse should be prepared to educate the client about which of the following medications that may be injected to help decrease the bladder hyperactivity?

Capsaicin, a specific C-fiber afferent neurotoxin. Feedback: Intravesical injection of medications, such as capsaicin and resiniferatoxin, that are specific C-fiber afferent neurotoxins may be used to decrease bladder hyperactivity. Botulinum toxin type A is used to produce paralysis of striated muscles of the external sphincter. Oxybutynin, an antimuscarinic agent, will decrease detrusor muscle tone. Urecholine, a cholinergic agonist, stimulates parasympathetic receptors to increase bladder tone.

A woman who is exhibiting clinical manifestations of a pituitary adenoma will likely complain of: Select all that apply.

Cessation of menses, Unusual milk secretion unrelated to pregnancy, and Infertility. Feedback: The signs and symptoms of pituitary adenomas include endocrine abnormalities related specifically to functional hormone-secreting adenomas and to the local mass effects from the expanding tumor. Lactotrophic adenomas are the most frequent type of hyperfunctioning pituitary adenoma. Hyperprolactinemia inhibits the pulsatile secretion of LH, which is essential for normal ovulation in women. Thus, manifestations of hyperprolactinemia are easily recognized to include amenorrhea (lack of menses), galactorrhea (spontaneous milk secretion unrelated to pregnancy), and infertility.

The most recent assessment of a client with a diagnosis of type 1 diabetes indicates a heightened risk of diabetic nephropathy. Which of the following assessment findings is most suggestive of this increased risk?

Microalbuminuria. Feedback: The increased glomerular filtration rate (GFR) that occurs in persons with early alterations in renal function is associated with microalbuminuria, which is an important predictor of future diabetic nephropathies. Hematuria is not directly suggestive of diabetic nephropathy, although it is a highly significant assessment finding. Orthostatic hypotension and diabetic retinopathy are not direct indicators of diabetic nephropathy.

Following the diagnosis of nephrotic syndrome, the nurse knows the clinical manifestations occur as a result of a decreased plasma colloidal osmotic pressure. Therefore, the nurse should assess the client for: Select all that apply.

Moist crackles in both lung fields, Areas of diminished breath sounds due to pleural effusions, Increased circumference in the abdomen related to fluid excess. Feedback: Generalized edema, which is a hallmark of the nephrotic syndrome, results from a decrease in the plasma colloidal osmotic pressure due to the hypoalbuminemia that develops as albumin is lost from the vascular compartment. Initially, the edema presents in dependent parts of the body such as the lower extremities, but becomes more generalized as the disease progresses. Dyspnea due to pulmonary edema, pleural effusions, and diaphragmatic compromise due to ascites (increase fluid in the abdominal cavity) can develop in persons with nephrotic syndrome. Live enlargement is not associated with nephrotic syndrome, but increased synthesis of lipoproteins in the liver secondary to a compensatory increase in albumin production may occur. Palpable kidney mass is associated with cancer.

As nitrogenous wastes increase in the blood, the CKD client may exhibit which of the following clinical manifestations? Select all that apply.

Numbness in lower extremities, Restless leg syndrome, and Pruritis. Feedback: The uremic state is characterized by signs and symptoms of altered neuromuscular function (e.g., fatigue, peripheral neuropathy, restless leg syndrome, sleep disturbances, uremic encephalopathy); gastrointestinal disturbances such as anorexia and nausea; white blood cell and immune dysfunction, and dermatologic manifestations such as pruritus. Photophobia and thrombocytopenia are usually not associated with CKD.

A drug abuser was found unconscious after shooting up heroin 2 days prior. Because of the pressure placed on the hip and arm, the client has developed rhabdomyolysis. The nurse knows this can:

Obstruct the renal tubules with myoglobin and damage tubular cells. Feedback: Myoglobin normally is not found in the serum or urine. It has a low molecular weight; if it escapes into the circulation, it is rapidly filtered in the glomerulus. A life-threatening condition known as rhabdomyolysis occurs when increasing myoglobinuria levels cause myoglobin to precipitate in the renal tubules, leading to obstruction and damage to surrounding tubular cells. Myoglobinuria most commonly results from muscle trauma but may result from exertion, hyperthermia, sepsis, prolonged seizures, and alcoholism or drug abuse. Rhabdomyolysis is not cured with anticoagulation administration nor does it cause kidney stones. Compartment syndrome occurs when there is insufficient blood supply to muscles and nerves due to increased pressure within one of the body's compartments.

Primary adrenal insufficiency is manifested by:

Serum sodium level of 120 mmol/L (low) and blood glucose level of 48 mg/dL (low). Feedback: Primary adrenal insufficiency is adrenal cortical hormone deficiency with elevated adrenocorticotropic hormone (ACTH) levels caused by a lack of feedback inhibition. Manifestations are related primarily to mineralocorticoid deficiency, causing increased urinary losses of sodium, chloride, and water, along with decreased excretion of potassium. The result is hyponatremia, loss of extracellular fluid, decreased cardiac output, and hyperkalemia. Because of a lack of glucocorticoid, the person with Addison disease has poor tolerance to stress. This deficiency causes hypoglycemia, lethargy, weakness, fever, and gastrointestinal symptoms such as anorexia, nausea, vomiting, and weight loss. Hypopigmentation results from elevated ACTH levels.

An 86-year-old female client has been admitted to the hospital for the treatment of dehydration and hyponatremia after she curtailed her fluid intake to minimize urinary incontinence. The client's admitting laboratory results are suggestive of prerenal failure. The nurse should be assessing this client for which of the following early signs of prerenal injury?

Sharp decrease in urine output. Feedback: Dehydration and its consequent hypovolemia can result in acute renal failure that is prerenal in etiology. The kidney normally responds to a decrease in GFR with a decrease in urine output. Thus, an early sign of prerenal injury is a sharp decrease in urine output. Postrenal failure is obstructive in etiology, and intrinsic (or intrarenal) renal failure is reflective of deficits in the function of the kidneys themselves.

In the intensive care unit (ICU), the nurse is caring for a trauma patient who has abdominal injuries that are beginning to have a decrease in BP and increased pulse rate and are pale with diaphoretic skin. The nurse is assessing the client for hemorrhagic shock. If the client is in shock, the nurse would expect to find:

Significant decrease in urine output due to decrease in renal blood flow: During periods of strong sympathetic stimulation, such as shock, constriction of the afferent arteriole causes a marked decrease in renal blood flow and thus glomerular filtration pressure. Consequently, urine output can fall almost to 0. Unless the injury is specific to the kidney, the client will not have blood in urine and urine production will not be excessive. Flank pain is associated with obstruction due to stone formation. The GFR will decrease rather than increase.

A child with Prader-Willi syndrome (PWS) has short stature, and the provider is considering treatment with growth hormone. Which aspect of this child's history should the nurse report to the provider?

Sleep apnea Growth hormone must be avoided in patients with PWS who are severely obese, who have a history of upper airway obstruction or sleep apnea, or who have severe respiratory impairment because of a risk of sudden death associated with these disorders. Behavior problems, low muscle tone, and mental impairment are commonly associated with PWS and are not contraindications to treatment with growth hormone.

When the glomerular transport maximum for a substance such as blood glucose is exceeded and its renal threshold has been reached, the substance will:

Spill into the urine. Feedback: When the substance (such as blood glucose) exceeds the number of carrier proteins available for transport, the transport maximum has been exceeded, the renal threshold is reached, and the substance will spill (not reabsorb) into the urine. Sodium cotransport helps to move the substance back into the tubule.

A patient who has been taking 25 mg of hydrocortisone each morning for several months reports feeling fatigued late in the day each day. What will the nurse tell the patient to discuss with the provider?

Splitting the daily dose into a morning and an afternoon dose. Patients generally take hydrocortisone once daily in the morning to mimic the body's natural cortisol release cycle. However, some patients develop fatigue late in the day. These patients may split the daily dose, taking two-thirds in the morning and one-third in the afternoon or early evening. Mineralocorticoids are given to maintain intravascular volume and regulate sodium, potassium, and hydrogen, so adding this drug would not help with fatigue. Fatigue is not a sign of steroid toxicity. Increasing the dose is not indicated.

A client has received an injection containing thyrotropin-releasing hormone (TRH) and is now being assessed for levels of thyroid-stimulating hormone (TSH). This client has undergone which of the following diagnostic tests?

Stimulation testing. Feedback: Introduction of TRH tests the pituitary gland's ability to produce TSH, and is an example of a stimulation test. Suppression testing examines a gland's response to a stimulus that would normally result in decreased hormone production. RIA and autoantibody testing are examples of direct and indirect measurement of serum levels of a hormone.

When hypofunction of an endocrine organ is suspected, which hormone test can be administered to measure and assess target gland response?

Stimulation. Feedback: Stimulating hormone can be given to identify (determine hypofunction) if the target gland is able to increase hormone response to increased stimulation. Agglutination with enzymes is a way of measuring hormone antigen levels. The 24-hour urine sample measures hormone metabolite excretion. Radioactive hormone-antibody binding levels are a method of measuring plasma levels.

A client who has been taking 80 mg of prednisone, a glucocorticoid, each day has been warned by his primary care provider to carefully follow a plan for the gradual reduction of the dose rather than stopping the drug suddenly. What is the rationale for this directive?

Stopping the drug suddenly may cause acute adrenal insufficiency. Feedback: Chronic suppression of the HPA system by the use of steroids causes atrophy of the adrenal gland, and the abrupt withdrawal of drugs can cause acute adrenal insufficiency. Activity of the HPA system is consequently insufficient. The efficacy of the drug is not the primary concern, and necrosis of the gland itself does not occur.

When describing to a newly diagnosed diabetic client how insulin is regulated, the nurse will draw upon her knowledge of which hormonal regulation mechanism?

The hypothalamic-pituitary-target cell system. Feedback: The hypophysis (pituitary plus hypothalamus) and hypothalamus stimulatory hormones regulate the release and synthesis of anterior pituitary hormones. The levels of hormones such as insulin and antidiuretic hormone (ADH) are regulated by feedback mechanisms that monitor substances such as glucose (insulin) and water (ADH) in the body. The levels of many of the hormones are regulated by feedback mechanisms that involve the hypothalamic-pituitary-target cell system.

Prior to undergoing diagnostic testing with contrast, it is recommended that older adult clients have their creatinine level checked. The rationale for this is to ensure the client:

Will not undergo an acute kidney injury by decreasing renal blood flow. Feedback: Some drugs, such as diuretics, high molecular weight radiocontrast media, the immunosuppressive drugs cyclosporine and tacrolimus, and the nonsteroidal anti-inflammatory drugs (NSAIDs), can cause acute kidney injury by decreasing renal blood flow. Checking creatinine levels do not predict the client's allergies, a kidney stone, or tolerance for stress testing.

A female teenager has experienced three uncomplicated urinary tract infections in the last 3 months. Knowing the anatomical location of the urethra, the nurse should educate this teenager about:

Wiping from front to back to prevent Escherichia coli contamination of the urethra. Feedback: Most commonly, urinary tract infections (UTIS) are caused by Escherichia coli that enter through the urethra. Escherichia coli are abundant in fecal matter. Other uropathic pathogens include Staphylococcus saprophyticus in uncomplicated UTIs and both non-E. coli gram-negative rods (Proteus mirabilis, Klebsiella pneumoniae, Pseudomonas) and gram-positive cocci (Staphylococcus aureus, group B Streptococcus) in complicated UTIS.

When explaining the body's compensatory mechanisms to maintain a normal pH, the health care provider knows that the renal system:

Works slower than the respiratory system, going into action 1 to 2 days after H+ remain elevated. Feedback: Only the kidney can eliminate hydrogen from the body. Virtually all the excess H+ excreted in the urine are secreted into the tubular fluid by means of tubular secretory mechanisms. The ability of the kidneys to excrete large amounts of H in the urine is accomplished by combining the excess ions with buffers in the urine. The three major urine buffers are HCO3 , phosphate (HPO42-), and ammonia (NH3). An important aspect of this buffer system is that the deamination process increases whenever the body's hydrogen ion concentration remains elevated for 1 to 2 days.

A nurse is teaching a group of parents about the role of testosterone in puberty for boys. To evaluate the group's understanding, the nurse asks, "What physiologic effects related to testosterone can you expect to see in your sons?" Which responses indicate an understanding of the role of testosterone in male puberty? (Select all that apply.)

a. "It promotes skeletal muscle growth." b. "It increases height and weight." e. "It causes acne." ANS: A, B, E The physiologic effects associated with androgen therapy are promotion of skeletal muscle growth, an increase in height and weight, acceleration of epiphyseal closure, deepening of the voice, oily skin, and acne. Androgen therapy accelerates epiphyseal closure; it does not delay it. Androgen therapy deepens the voice rather than raising the pitch.

Nitrofurantoin [Macrodantin] is prescribed for an adolescent female patient with acute cystitis. What should the nurse include in the teaching for this patient? (Select all that apply.)

a. "Make sure you tell your prescriber if you might be pregnant." b. "If you experience any tingling or numbness, stop taking the drug and call the clinic immediately." c. "Headaches and drowsiness can occur and are mild side effects." e. "Your urine may have a brown tinge while you are taking this drug." ANS: A, B, C, E Nitrofurantoin is linked to serious birth defects and is not recommended during pregnancy. Tingling and numbness indicate neuropathy, which is a serious and often irreversible side effect. Central nervous system (CNS) side effects usually are mild and reversible. Sulfonamides cannot be taken with methenamine, because they can cause crystalluria. Patients' urine may have a brown tinge while they are taking this drug.

Which are indications for discontinuing growth hormone (GH) therapy in children with documented growth hormone deficiency? (Select all that apply.)

a. Epiphyseal closure has begun. b. A satisfactory adult height has been achieved. e. The child no longer responds to the hormone. ANS: A, B, E Treatment with GH for a child with a documented GH deficiency may continue until a satisfactory adult height has been achieved, if a child does not show a response to the hormone, or until epiphyseal closure begins. Treatment does not continue for life, and GH should not be administered during or after closure of the epiphyses. Treatment does not depend on growth hormone levels; these are measured to ensure therapeutic dosing.

Which are contraindications for taking sildenafil [Viagra]? (Select all that apply.)

a. Patients who are taking nitroglycerin for angina pectoris b. Patients with nonarteritic ischemic optic neuropathy (NAION) in one eye c. Patients with BPH who are taking alphaj-adrenergic antagonists ANS: A, B, C Men who take nitroglycerin should not take sildenafil, because at least 24 hours should elapse between taking nitroglycerin and taking sildenafil. Because sildenafil is associated with the development of NAION, patients with that disorder in one eye should not take sildenafil. Patients with low blood pressure should use caution in taking sildenafil, but it is not contraindicated. Patients who have had an MI in the past 6 months should use caution in taking sildenafil.

A patient who has chronic adrenal insufficiency is admitted to the hospital for an open cholecystectomy. The nurse obtaining the admission history learns that the patient takes hydrocortisone 25 mg PO daily in the morning. The patient's surgery is scheduled for the next morning. The nurse will expect an order to:

administer hydrocortisone 50 mg IV before surgery. Patients who take steroids need extra steroid before situations that cause stress, such as surgery. Failure to administer the increased dose can prove fatal. For surgeries that cause moderate stress, such as a cholecystectomy, patients should be given 50 mg of hydrocortisone intravenously the day of the procedure, followed by a taper over 1 to 2 days to the usual replacement dose. Giving the usual dose or giving an increased oral dose is not indicated.

A patient with a history of benign prostatic hyperplasia is admitted to the unit. The patient is taking multiple medications, including terazosin [Hytrin). During the evening rounds, the patient begins to complain of dizziness and nasal congestion. Upon assessment, the patient is somnolent and has a blood pressure of 101/42 mm Hg. The nurse is correct to suspect:

adverse effects. The principal adverse effects of terazosin and doxazosin are hypotension, fainting, dizziness, somnolence, and nasal congestion. No evidence indicates priapism, sinus infection, or an allergic reaction.

A patient has a positive urine culture 1 week after completion of a 3-day course of antibiotics. The nurse anticipates that the prescriber will:

begin a 2-week course of antibiotics. Patients who develop a subsequent urinary tract infection after treatment are treated in a stepwise fashion, beginning with a longer course of antibiotics. The next steps would be to begin a 4- to 6 week course of therapy, followed by a 6-month course of therapy if that is unsuccessful. If urinary tract infections are thought to be caused by other complicating factors, an evaluation for structural abnormalities may be warranted. Unless the infections are severe or are complicated, intravenous antibiotics are not indicated.

A patient with a history of renal calculi has fever, flank pain, and bacteriuria. The nurse caring for this patient understands that it is important for the provider to:

begin antibiotic therapy after urine culture and sensitivity results are available Patients with renal calculi are more likely to have complicated urinary tract infections that have less predictable microbiologic etiologies. Because the symptoms are mild, it is important first to obtain a culture and sensitivity to assist with antibiotic selection. If symptoms worsen, a broad spectrum antibiotic may be started until sensitivity information is available. Intravenous antibiotics are indicated for severe pyelonephritis. Long-term prophylaxis is not indicated unless this patient develops frequent reinfection.

A 1-year-old child with cretinism has been receiving 8 mcg/kg/day of levothyroxine [Synthroid). The child comes to the clinic for a well-child checkup. The nurse will expect the provider to:

change the dose of levothyroxine to 6 mcg/kg/day. In the treatment of cretinism, thyroid dosing decreases with age. For infants 6 to 12 months of age, the dose is 6 mcg/kg/day. At 1 year of age, the dose is reduced to 5 to 6 mcg/kg/day. For all children, treatment should continue for 3 years. It is incorrect to increase the dose with age. After 3 years of therapy, the patient undergoes a trial of 4 weeks without the drug, followed by assessment of the TSH and T4 levels, to determine whether the drug may be discontinued.

A student nurse is taking a test on the endocrine system. From the following list of clinical manifestations, she needs to select the ones she would see in hypothyroidism. Which answers should she select? Select all that apply.

Weight gain despite loss of appetite, Coarse brittle hair, and Puffy face with swollen eyelids. Feedback: The hypometabolic state associated with hypothyroidism is characterized by a gradual onset of weakness and fatigue, a tendency to gain weight despite a loss of appetite, and cold intolerance. As the condition progresses, the skin becomes dry and rough and the hair becomes coarse and brittle. Reduced conversion of carotene to vitamin A and increased blood levels of carotene may give the skin a yellowish color. The face becomes puffy with edematous eyelids, and there is thinning of the outer third of the eyebrows. Nervousness with fine muscle tremors and heat intolerance are signs of hyperthyroidism.

A female client asks, "Why do I leak urine every time I cough or sneeze?" The health care worker's response is based on which physiologic principle?

When intravesical pressure exceeds maximal urethral closure pressure. Feedback: Stress incontinence represents the involuntary loss of urine that occurs when, in the absence of detrusor muscle action, the intravesical pressure exceeds the maximum urethral closure pressure. Stress incontinence, which is a common problem in women of all ages, occurs as the result of weakness or disruption of pelvic floor muscles, leading to poor support of the vesicourethral sphincters. Except during the act of micturition, intraurethral pressure is normally greater than intravesical pressure. Urge incontinence and overactive bladder are associated with urgency caused by bladder infection or CNS or myogenic mechanisms. Overflow incontinence is an involuntary loss of urine that occurs when intravesical pressure exceeds the maximal urethral pressure because of bladder distention in the absence of detrusor activity.

Which of the following individuals is experiencing the effects of a primary endocrine disorder? A client:

Who has low calcium levels because of the loss of his parathyroid gland. Feedback: The loss of a gland, and the subsequent absence of the hormone that it normally produces, results in a primary endocrine disorder. The lack of a stimulating hormone such as ACTH or TSH results in a secondary disorder, whereas hypothalamic dysfunction causes tertiary endocrine disorders.

Which of the following clinical manifestations lead the health care worker to suspect the client is at the end-stage expression of hypothyroidism? A client: Select all that apply.

Who takes analgesics for chronic pain that goes into a coma and Brought to the emergency department with hypothermia who presents with low serum sodium levels. Feedback: Myxedematous coma is a life-threatening, end-stage expression of hypothyroidism. It is characterized by coma, hypothermia, cardiovascular collapse, hypoventilation, and severe metabolic disorders including hyponatremia, hypoglycemia, and lactic acidosis. With the hypermetabolic state of hyperthyroidism, there are frequent complaints of nervousness, irritability, and fatigability. Weight loss is common despite a large appetite. Other manifestations include tachycardia, palpitations, shortness of breath, excessive sweating, muscle cramps, and heat intolerance.

A pregnant female patient with bacteriuria, suprapubic pain, urinary urgency and frequency, and a low-grade fever is allergic to sulfa, ciprofloxacin, and amoxicillin. The nurse knows that the best alternative for treating this urinary tract infection is with:

methenamine [Hiprex]. Methenamine is an excellent second-line drug for this patient and is indicated because of the patient's multiple drug sensitivities. It is safe in pregnancy, and there is no drug resistance. Nitrofurantoin has potential harmful effects on the fetus and should not be used during pregnancy. Single-dose regimens are not recommended in pregnant women. Cephalexin can have cross-reactivity with amoxicillin.

A 7-year-old child who is otherwise healthy is receiving mecasermin [Increlex] replacement therapy to treat severe primary deficiency of insulin-like growth factor-1 (IGF-1). The child develops tonsillar hypertrophy. The nurse anticipates that the provider will recommend:

tonsillectomy. Mecasermin can cause hypertrophy of the tonsils, which can be managed by tonsillectomy if needed. Antibiotics are not indicated, because the tonsil enlargement is not caused by infection. Neither reducing the dose of mecasermin nor discontinuing the drug is indicated.

A 50-year-old postmenopausal patient who has had a hysterectomy has moderate to severe vasomotor symptoms and is discussing estrogen therapy (ET) with the nurse. The patient is concerned about adverse effects of ET. The nurse will tell her that:

transdermal preparations have fewer side effects. Transdermal preparations of estrogen have fewer adverse effects, use lower doses of estrogen, and have less fluctuation of estrogen levels than do oral preparations. Progesterone is contraindicated in women who have undergone hysterectomy. Intravaginal preparations are most useful for treating local estrogen deficiency such as vaginal and vulvar atrophy. Side effects of ET are the same at the patient's age as for other women using ET. Hepatotoxicity is an ever-present risk in athletes who abuse androgens. In female patients, androgens can cause menstrual irregularities. Breast size will decrease. Virilization effects may not be reversible after the drug is stopped.

What are the effects of prolactin secretion in males? (Select all that apply.)

b. Decreased libido c. Delayed puberty d. Galactorrhea ANS: B, C, D Prolactin can cause decreased libido, delayed puberty, and galactorrhea in males, but it does not cause breast development or infertility.

A diabetic client is controlled on Avandia (rosiglitazone), a thiazolidinedione medication that acts at the level of nuclear peroxisome proliferator-activated receptors (PPARs) to promote:

A) Glucose uptake. Feedback: The thiazolidinedione medications, which are used in the treatment of type 2 diabetes mellitus, act at the level of nuclear PPAR-y receptors to promote glucose uptake and utilization by adipose tissue cells. These drugs do not increase release of insulin from the pancreas, increase BMR, or promote weight loss.

When educating a client with possible glucocorticoid dysfunction, the nurse will explain that the CRH controls the release of ACTH. The best time to perform the blood test to measure peak ACTH levels would be:

A) 06:00 to 08:00 AM. Feedback: Levels of cortisol increase as ACTH levels rise and decrease as ACTH levels fall. There is considerable diurnal variation in ACTH levels, which reach their peak in the early morning (around 6 to 8 AM) and decline as the day progresses.

Which of the following residents of a long-term facility is exhibiting clinical manifestations of hypothyroidism?

A) An 80-year-old woman who has uncharacteristically lost her appetite and often complains of feeling cold. Feedback: Loss of appetite and cold intolerance are characteristic symptoms of hypothyroidism. Arrhythmias, agitation, and infections are not typically associated with hypofunction of the thyroid gland.

Paracrine action involves which of the following characteristics?

Act locally on cells other than those that produce the hormone. Feedback: When hormones act locally on cells other than those that produced the hormone, the action is called paracrine. Paracrine action is not synonymous with autoregulation, and action on the same cells that produced the hormone is autocrine action.

Cyclic adenosine monophosphate (CAMP) performs which of the following roles in the functioning of the endocrine system?

Acting as a second messenger to mediate hormone action on target cells. Feedback: CAMP is one of the most common second messengers, whose role is to generate an intracellular signal in response to cell surface receptor activation by a hormone. CAMP does not mediate hormone synthesis, act as a receptor itself, or inactivate hormones.

A patient has been taking levothyroxine for several years and reports that "for the past 2 weeks, the drug doesn't seem to work as well as before." What will the nurse do?

Ask the patient when the prescription was last refilled. Not all levothyroxine preparations have the same drug bioavailability; therefore, if a patient is experiencing differing effects, the pharmacist may have switched brands. Asking a patient about a recent refill may help to explain why the drug has different effects. An elevated temperature and tachycardia would be signs of toxicity, not of a decrease in effectiveness. Calcium supplements and food would only interfere with absorption and further reduce the drug's effectiveness.

A 33-year-old client has been admitted to the hospital for the treatment of Graves disease. Which of the following assessments should the client's care team prioritize?

Assessment of the client's vision and oculomotor function/ Feedback: Ophthalmopathy occurs in a large proportion (up to one third) of clients with Graves disease and may result in permanent vision damage. This supersedes the importance of cardiac, neurologic, and peripheral vascular assessments, although these assessments are relevant to the broader effects of hyperthyroidism that the client may likely experience.

increasing in both frequency and severity. The health care worker should explain which of the following physiological factors to the client that is likely contributing to recurrent UTIS?

Reflux flow of urine that can occur from coughing or sneezing Feedback: A phenomenon called ureterovesical reflux occurs when urine from the urethra moves into the bladder. In women, ureterovesical reflux can occur during activities such as coughing or squatting, in which an increased intra-abdominal pressure causes the urine to be squeezed into the urethra and then to flow back into the bladder as the pressure decreases. Reflux flow of urine is a significant risk factor for UTIs in general and for recurrent UTIs in particular. Fluctuations in urine pH are not noted to contribute to recurrent UTIs. Urethral trauma and inadequate fluid intake may contribute to the development or prolonging of UTIs, but these risk factors are less significant than the presence of urine reflux.

Which of the following clinical manifestations following thyroidectomy would alert the nurse that the client is going into a life-threatening thyroid storm? Select all that apply.

Temperature of 104.2°F, Telemetry showing heart rate of 184, and Extremely agitated. Feedback: Thyroid storm, or crisis, is an extreme and life-threatening form of thyrotoxicosis, rarely seen today. When it does occur, it is seen most often in undiagnosed cases or in person with hyperthyroidism that has not been adequately treated. It often is precipitated by stress such as an infection, diabetic ketoacidosis, physical or emotional trauma, or manipulation of a hyperactive thyroid gland during thyroidectomy. It is manifested by a very high fever, extreme cardiovascular effects (tachycardia, HF, angina), and severe CNS effects (agitation, restlessness, and delirium).

An automobile accident client is brought to the emergency department in hypovolemic shock from internal bleeding. Nurses are closely monitoring urine output since a significant decrease signifies that...

The SNS has caused afferent arteries to constrict to decrease blood flow. Feedback: The afferent and the efferent arterioles are innervated by the sympathetic nervous system and are sensitive to vasoactive hormones, such as angiotensin II. During periods of strong sympathetic stimulation, such as shock, constriction of the afferent arteriole causes a marked decrease in renal blood flow and thus glomerular filtration pressure. Consequently, urine output can fall almost to 0. There is not enough information to conclude the kidneys are injured, the renal arteries are clotted, or the vagus nerve has been innervated.

A patient received atropine intravenously before surgery. The recovery room nurse notes that the patient is delirious upon awakening and has a heart rate of 96 beats/min, a respiratory rate of 22 breaths per minute, and a blood pressure of 110/78 mm Hg. The nurse notifies the anesthesiologist, who will order:

an acetylcholinesterase inhibitor to compete with the antimuscarinic agent at receptors. This patient is showing signs of antimuscarinic toxicity caused by the atropine given during surgery. The most effective antidote is physostigmine, which inhibits acetylcholinesterase, allowing acetylcholine to build up at cholinergic junctions and compete with the antimuscarinic agent for receptor binding. Activated charcoal is useful only if an antimuscarinic agent has been ingested because it impedes absorption from the GI tract. Because this patient's psychotic symptoms are caused by an antimuscarinic agent, physostigmine should be given to treat the cause; an antipsychotic medication would only treat the symptom. Ipratropium bromide is an antimuscarinic agent and would only compound the effects. This patient's respiratory rate is only mildly elevated.

A male patient tells the nurse he awakens once or twice each night to void and has difficulty starting his stream of urine. He describes these symptoms as "annoying." The patient's provider examines him and notes that the prostate is moderately enlarged. The patient is sexually active and tells the nurse that he does not want to take any medication that will interfere with sexual function. The nurse anticipates the provider will order:

doxazosin [Cardura]. Nonselective alphaj-adrenergic antagonists do not commonly affect sexual function and are useful in patients with mild to moderate symptoms, so doxazosin would be a drug of choice for this patient. Finasteride is a 5-alpha-reductase inhibitor and is used for patients with more severe enlargement of the prostate; it also reduces ejaculate volume and libido. Silodosin is a selective alpha1-adrenergic antagonist and can cause abnormal ejaculation. Transurethral prostatectomy is reserved for patients with more severe enlargement of the prostate.

A client who has had recurrent UTIs asks the nurse about the old wise tale of drinking cranberry juice daily. The nurse can respond:

"Research suggests cranberry juice will reduce bacterial adherence to the lining of the urinary tract." Feedback: Cranberry juice or blueberry juice has been suggested as a preventive measure for persons with frequent UTIs. Studies suggest that these juices reduce bacterial adherence to the epithelial lining of the urinary tract.

A nurse is explaining congenital adrenal hyperplasia (CAH) to a group of nursing students. Which statement by a student indicates understanding of the teaching?

"CAH is the result of an inability to synthesize glucocorticoids." CAH results from an inborn deficiency of the enzymes needed for glucocorticoid synthesis. The condition is marked by increased production of ACTH, because the pituitary attempts to enhance glucocorticoid synthesis. Newborn screening indicates a deficiency of 21-alpha-hydroxylase; if this is noted, follow-up testing must be done to confirm a diagnosis of CAH. The enzyme 21 alpha-hydroxylase increases the production of glucocorticoids. When this enzyme is deficient, androgens are produced in excess because of stimulation by ACTH.

The nurse is scheduled to teach a client experiencing urinary incontinence about Kegel exercises. Which of the following descriptors should the nurse include in this education?

"Contract and relax the pelvic floor muscles at least 10 times every hour while awake." Feedback: Exercises for the pelvic muscles or Kegel exercises involve repetitive contraction and relaxation of the pelvic floor muscles and are an essential component of client-dependent behavioral interventions. None of the other distractors are examples of Kegel exercises.

The nurse is providing patient education about glucocorticoid therapy to a patient preparing to be discharged home. Which statement made by the patient best demonstrates understanding of glucocorticoid therapy?

"I may take two-thirds of the dose in the morning and one-third in the afternoon." Patients should be instructed to follow the prescribed dosing schedule. Some prescribers recommend dividing the daily dose by taking two-thirds in the morning and one-third in the evening. Other prescribers recommend taking the entire daily dose at bedtime. Taking the dose at breakfast, dividing the dose in half, and taking a dose with each meal are not recommended dosing schedules.

A neighbor is complaining to a friend (who happens to be a nurse) about several changes in their body. Which of the following complaints raises a "red flag" because it could be a sign of epithelial cell bladder cancer?

"I noticed my urine is pinkish red, but I'm not having any pain when I pee." Feedback: The most common sign of bladder cancer is intermittent painless hematuria. Fluid retention, stress incontinence, and pain with exercise are not usual signs of cancer.

A nurse is teaching a patient who will begin taking methimazole [Tapazole) for Graves' disease about the medication. Which statement by the patient indicates understanding of the teaching?

"I should report a sore throat or fever to my provider if either occurs." Agranulocytosis is rare but can occur with methimazole, so patients should report signs of infection, such as a sore throat or fever. Liver toxicity is not a side effect, so liver function tests are not indicated. Because agranulocytosis often develops rapidly, periodic blood counts do not guarantee early detection. Methimazole is contraindicated in the first trimester of pregnancy.

The nurse is providing patient education to a patient who will begin taking fludrocortisone [Florinef as adjunctive therapy to hydrocortisone. Which statement by the patient indicates understanding of the teaching?

"I should report any swelling of my hands and feet." Fludrocortisone is a mineralocorticoid that regulates sodium, potassium, and water balance. Water and sodium retention is a particular concern, so the patient should be taught to report any signs of fluid retention, such as swelling of the hands and feet. Fludrocortisone elevates the blood pressure, so hypotension is not a concern. Because fluid retention causes weight gain, patients should be taught to report any increase in weight. Patients should report decreased urine output.

Tadalafil (Cialis) was prescribed 4 weeks ago for a patient with erectile dysfunction. The patient also takes prazosin [Minipress] for hypertension. Which statement by the patient best demonstrates understanding of the use of tadalafil (Cialis)?

"I should take this medication no more than once a day." Tadalafil should be taken no more than once a day and without regard to meals. Tadalafil has a half-life of 17.5 hours and should not be taken twice a day. Men with moderate renal or hepatic insufficiency should take a reduced dosage. This patient may be confusing the actions of tadalafil and prazosin.

A nurse is providing teaching for a nondiabetic adult who develops growth hormone deficiency and who will begin treatment with somatropin [Humatrope]. Which statement by the patient indicates understanding of the teaching?

"I will need to monitor my blood pressure frequently while taking this drug." Growth hormone in adults causes an increase in systolic blood pressure, so patients should be taught to monitor blood pressure while taking the drug. Subcutaneous dosing is as effective as IM dosing and is preferred, because it is less painful. Although muscle mass will increase, strength and height will not. Growth hormone is diabetogenic but causes significant problems in patients with preexisting diabetes.

A patient will begin using a transdermal preparation of a muscarinic antagonist for overactive bladder (OAB). The nurse teaches the patient what to do if side effects occur. Which statement by the patient indicates the need for further teaching?

"I will take Benadryl for any itching caused by a local reaction to the patch." Benadryl is an antihistamine, and even though it is not classified as a muscarinic antagonist, it has anticholinergic effects. Giving it with a muscarinic antagonist greatly enhances these effects, so it should not be used. Muscarinic antagonists cause dry mouth, and patients should be taught to use sugar-free gum or candies to help with this. Muscarinic antagonists can cause constipation, and laxatives may be used. Medication applied topically can be transferred to others who come in contact with the skin, so the site should be covered.

A client with a recent diagnosis of renal failure who will require hemodialysis is being educated in the dietary management of the disease. Which of the client's following statements shows an accurate understanding of this component of treatment?

"I've made a list of high-phosphate foods, so that I can try to avoid them." Feedback: Persons with chronic kidney disease (CKD) are usually encouraged to limit their dietary phosphorus as a means of preventing secondary hyperparathyroidism, renal osteodystrophy, and metastatic calcification. Excessive potassium and fluids are likely contraindicated in kidney disease individuals require hemodialysis. The amount of dietary fiber intake is not a priority when looking at primary needs of a CKD patient's food intake.

A nurse is teaching a patient who has been diagnosed with hypothyroidism about levothyroxine [Synthroid). Which statement by the patient indicates a need for further teaching?

"If I take calcium supplements, I may need to decrease my dose of Synthroid." Patients taking calcium supplements should take these either 4 hours before or after taking levothyroxine, because they interfere with levothyroxine absorption. Many heartburn medications contain calcium, so patients should consult their provider before taking them. Insomnia, tremors, and tachycardia are signs of levothyroxine toxicity and should be reported. Iron also interferes with levothyroxine absorption, so dosing should be 4 hours apart.

A nurse is teaching a patient about the use of sildenafil [Viagra] for erectile dysfunction. Which statement by the patient indicates understanding of the teaching?

"If my erection lasts longer than 4 hours, I should contact my provider." Priapism can occur and can cause tissue damage to the penis, so an erection lasting longer than 4 hours should be reported. Patients who use nitroglycerin should not use sildenafil within 24 hours of taking this drug. Patients should be advised to take sildenafil 30 minutes to 4 hours before anticipated sexual activity. Sildenafil does not cause an erection without sexual stimuli.

A healthy male patient who does not have erectile dysfunction asks about medications to improve sexual stamina. What will the nurse tell this patient?

"Medications for ED have no effect on erections in healthy men." Medications for ED will have little or no effect on erection quality or duration in otherwise healthy men who do not have ED. These medications will not cause priapism in otherwise healthy men.

A public health nurse is conducting a health promotion class for a group of older adults. Which of the participants' following statements demonstrates an accurate understanding of the risk factors for bladder cancer?

"More than ever, I guess it would worthwhile for me to quit smoking." Feedback: Smoking is implicated in 30% to 50% of all bladder cancers among males who are current or past smokers. Cranberry juice may be of benefit in the prevention not cancer, and neither poor diet nor family history is as significant as cigarette smoking in the etiology of bladder cancer.

In anatomy class, the instructor asks, "Explain how urine is expelled from the bladder during voiding." The student with the most accurate response would be:

"The detrusor muscle contract down on the urine and the ureteral orifices are forced shut. The external sphincter relaxes as urine moves out of the bladder." Feedback: During the act of micturition, the detrusor muscle of the bladder fundus and bladder neck contracts down on the urine and the ureteral orifices are forced shut. The bladder neck is widened and shortened, and the external sphincter relaxes as urine moves out of the bladder. Descent of the diaphragm and contraction of the abdominal muscles raise intra-abdominal pressure and aid in the expulsion of urine from the bladder.

A pregnant patient with fever, flank pain, and chills has a history of two previous bladder infections before getting pregnant. She is allergic to several antibiotics. She reports having taken methenamine successfully in the past. What will the nurse tell her?

"This agent is not effective against infections of the upper urinary tract." Methenamine is safe for use during pregnancy and would be an excellent choice for this patient if she had a lower urinary tract infection. However, it is not an effective agent for upper urinary tract infection, because it is a prodrug that must break down into ammonia and formaldehyde to be effective. There is not enough time for formaldehyde to form in the kidneys, so it is not effective in the upper tract. Nitrofurantoin is linked to serious birth defects and also must be given with food to prevent gastrointestinal problems.

The nurse prepares a patient with Graves' disease for radioactive iodine (1311) therapy. Which statement made by the patient best demonstrates understanding of 1311 therapy?

"This drug will be taken up by the thyroid gland and will destroy the cells to reduce my hyperthyroidism." Iodine-131 can be used to destroy thyroid tissue in patients with hyperthyroidism; no further teaching is necessary. The patient does not need to isolate himself from others. The treatment will not reduce intolerance to cold, nor will it affect weight gain. The patient will not need the treatment daily.

Which of the following clients are at risk for developing hypothyroidism? Select all that apply.

A client who is prescribed amiodarone for frequent dysrhythmias, A client who has precancerous thyroid lesions who underwent ablation with radiation, A female experiencing an autoimmune disorder called thyroiditis, and A bipolar client prescribed lithium carbonate. Feedback: The most common cause of hypothyroidism is Hashimoto thyroiditis, an autoimmune disorder in which the thyroid gland may be totally destroyed by an immunologic process. It is the major cause of goiter and hypothyroidism in children and adults. Hypothyroidism may result from thyroidectomy (i.e., surgical removal) or ablation of the gland with radiation. Certain goitrogenic agents, such as lithium, and the antithyroid drugs propylthiouracil and methimazole in continuous dosage can block hormone synthesis and produce hypothyroidism with goiter. Large amounts of iodine (i.e., ingestion of kelp tablets or iodide-containing cough syrups, or administration of iodide-containing radiographic contrast media or the cardiac drug amiodarone, which contains 75 mg of iodine per 200-mg tablet) also can block thyroid hormone production and cause goiter, particularly in persons with autoimmune thyroid disease. Myxedema is associated with severe hypothyroidism and is characterized by a non pitting mucus-type edema caused by the accumulation of hydrophobic extracellular matrix substances in the connective tissues of a number of body tissues. Although the myxedema is most obvious in the face and other superficial parts, it also affects many of the body organs and is responsible for many of the manifestations of the hypothyroid state.

Which of the following individuals likely faces the greatest risk for the development of chronic kidney disease?

A client with a recent diagnosis of type 2 diabetes who does not monitor his blood sugars or control his diet. Feedback: Chronic kidney disease (CKD) is a pathophysiologic process that results in the loss of nephrons and a decline in renal function that has persisted for more than 3 months. CKD can result from diabetes, hypertension, glomerulonephritis, lupus (SLE), and polycystic kidney disease. The prevalence and incidence of CKD continue to grow, reflecting the growing elderly population and the increasing number of people with diabetes and hypertension. Hemorrhage may result in acute renal failure, but it is not associated with chronic kidney disease. Stroke and loss of the thyroid gland are not noted to underlie cases of chronic kidney disease.

Which of the following clients is benefiting from the renin-angiotensin-aldosterone Mechanism?

A college student admitted to the neurotrauma unit following traumatic brain injury requiring surgery to evacuate a large hematoma. Feedback: The kidney releases renin, which enters the bloodstream to convert angiotensinogen to angiotensin I. The angiotensin I travels to the lungs, where it is converted to angiotensin II. Angiotensin II acts directly on the kidneys. Renin functions by means of angiotensin II to produce intrarenal vasoconstriction. This helps to regulate blood pressure, which could be a problem for the client having bleeding (hematoma) inside the brain.

When explaining urinalysis results that show the presence of cast cells, the nurse informs the client that casts cells develop when the client has: Select all that apply.

A high protein concentration of the urine. High urine osmolality. Feedback: Casts are molds of the distal nephrons' lumen. A gel-like substance (Tamm-Horsfall mucoprotein) is formed in the tubular epithelium and is the major protein constituent of urinary casts. Casts composed of this gel but devoid of cells are called hyaline casts. These casts develop when the protein concentration of the urine is high, urine osmolality is high, and urine pH is low.

The primary care provider for a newly admitted hospital client has added the glomerular filtration rate (GFR) to the blood work scheduled for this morning. The client's GFR results return as 50 mL/minute/1.73 m2. The nurse explains to the client that this result represents:

A loss of over half the client's normal kidney function. Feedback: In clinical practice, GFR is usually estimated using the serum creatinine concentration. A GFR below 60 mL/minute/1.73 m2 represents a loss of one half or more of the level of normal adult kidney function. The GFR is not diagnostic for concentrated urine or the need to drink more water.

Which of the following statements about the use of angiotensin-converting enzyme inhibitor medications and autosomal recessive polycystic kidney disease (ARPKD) is accurate?

ACE inhibitors may interrupt the renin-angiotensin-aldosterone system to reduce renal vasoconstriction. Feedback: In addition to increasing water intake to decrease vasopressin levels, the angiotensin-converting enzyme (ACE) inhibitors or angiotensin II receptor blockers (ARBs) may be used to interrupt the renin-angiotensin-aldosterone system as a means of reducing intraglomerular pressure and renal vasoconstriction. Although not approved by the Food and Drug Administration (FDA), there has been recent interest in the use of vasopressin receptor antagonists (vaptans) to decrease cyst development.

A lung cancer client with small cell carcinoma may secrete an excess of which hormone causing an ectopic form of Cushing syndrome due to a non pituitary tumor?

ACTH. Feedback: The third form (of Cushing syndrome) is ectopic Cushing syndrome, caused by a non pituitary ACTH-secreting tumor. Certain extra pituitary malignant tumors such as small cell carcinoma of the lung may secrete ACTH or, rarely, CRH and produce Cushing syndrome. The adrenal sex hormone dehydroepiandrosterone (DHEA) contributes to the pubertal growth of body hair, particularly pubic and axillary hair in women. Thyroid-stimulating hormone (TSH) levels are used to differentiate between primary and secondary thyroid disorders. Although secretion of growth hormone (GH) has diurnal variations over a 24-hour period, with nocturnal sleep bursts occurring 1 to 4 hours after onset of sleep, it is unrelated to ACTH and/or CRH secretion.

Which of the following individuals displays the precursors to acromegaly?

An adult with an excess of growth hormone due to an adenoma. Feedback: When growth hormone (GH) excess occurs in adulthood or after the epiphyses of the long bones have fused, it causes a condition called acromegaly, which represents an exaggerated growth of the ends of the extremities.

A child is recovering from a bout with group A B-hemolytic Streptococcus infection. They return to the clinic a week later complaining of decrease in urine output with puffiness and edema noted in the face and hands. The health care provider suspects the child has developed:

Acute postinfectious glomerulonephritis. Feedback: The classic case of poststreptococcal glomerulonephritis follows a streptococcal infection by approximately 7 to 12 days—the time needed for the development of antibodies. Oliguria, which develops as the GFR decreases, is one of the first symptoms. Proteinuria and hematuria follow because of increased glomerular capillary wall permeability. Sodium and water retention gives rise to edema (particularly of the face and hands) and hypertension. Adults with medullary cystic kidney disease present first with polyuria, polydipsia, and enuresis (bed-wetting), which reflect impaired ability of the kidneys to concentrate urine. The typical infant with ARPKD presents with bilateral flank masses, accompanied by severe renal failure, signs of impaired lung development, and variable degrees of liver fibrosis and portal hypertension. Acute nephritic syndrome is characterized by sudden onset of hematuria, variable degrees of proteinuria, diminished glomerular filtration rate (GFR), oliguria, and signs of impaired renal function.

The physician suspects that a client with kidney stones has developed magnesium ammonium phosphate (struvite) stones based on which of the following urinalysis results? Select all that apply.

Alkaline urine pH, High urine phosphate level, and High bacterial count. Feedback: Magnesium ammonium phosphate stones, also called struvite stones, form only in alkaline urine and in the presence of bacteria that possess an enzyme called urease, which splits the urea in the urine into ammonia and carbon dioxide. The ammonia that is formed takes up a hydrogen ion to become an ammonium ion, increasing the pH of the urine so that it becomes more alkaline. Because phosphate levels are increased in alkaline urine and because magnesium always is present in the urine, struvite stones form. Uric acid stones develop in conditions of gout and high concentrations of uric acid in the urine. Cystine stones account for less than 1% of kidney stones overall but represent a significant proportion of childhood calculi. They are seen in cystinuria, which results from a genetic defect in renal transport of cystine.

A client has been given the diagnosis of diffuse glomerulonephritis. They ask the nurse what diffuse means. The nurse responds:

All glomeruli and all parts of the glomeruli are involved. Feedback: Glomerular changes can be diffuse, involving all glomeruli and all parts of the glomeruli; focal, meaning only some of the glomeruli are affected; segmental, involving only a certain segment of each glomerulus; and mesangial, affecting only mesangial cells.

A client who has developed stage 3 renal failure has been diagnosed with high phosphate levels. To avoid the development of osteodystrophy, the physician may prescribe a phosphate-binding agent that does not contain:

Aluminum. Feedback: Aluminum-containing antacids can contribute to the development of osteodystrophy, whereas calcium-containing phosphate binders can lead to hypercalcemia, thus worsening soft tissue calcification, especially in persons receiving vitamin D therapy. Sevelamer hydrochloride is a newer phosphate-binding agent that does not contain calcium or aluminum.

Hormones are usually divided into categories according to their structure. The release of epinephrine would be classified as:

Amines and amino acids. Feedback: Hormones are divided into three categories according to their structures: amines and amino acids; polypeptides, proteins, and glycoproteins; and steroids. The amine and amino acid hormones include norepinephrine and epinephrine, which are derived from a single amino acid (i.e., tyrosine). The peptide, polypeptide, protein, and glycoprotein hormones can be as small as thyrotropin-releasing hormone (TRH), which contains three amino acids, and as large, and as large and complex as growth hormone (GH) and follicle-stimulating hormone (FSH). Steroid hormones, such as the glucocorticoids, are derivatives of cholesterol.

Which patient with a urinary tract infection will require hospitalization and intravenous antibiotics?

An older adult man with a low-grade fever, flank pain, and an indwelling catheter. The patient with an indwelling catheter and signs of pyelonephritis shows signs of a complicated UTI, which is best treated with intravenous antibiotics. The other three patients show signs of uncomplicated urinary tract infections that are not severe and can be treated with oral antibiotics.

A client has a diagnosis of chronic renal failure secondary to diabetic nephropathy. Which of the following hematologic changes may result from this client's kidney disorder?

Anemia. Feedback: Erythropoietin is a polypeptide hormone that regulates the differentiation of RBCs in the bone marrow. Between 89% and 95% of erythropoietin is formed in the kidneys. The synthesis of erythropoietin is stimulated by tissue hypoxia, which may be brought about by anemia, residence at high altitudes, or impaired oxygenation of tissues due to cardiac or pulmonary disease. Persons with chronic kidney disease often are anemic because of an inability of the kidneys to produce erythropoietin. Changes in platelet or white blood cell levels are not likely to result directly from renal failure.

A client with a long-standing diagnosis of chronic kidney disease has been experiencing increasing fatigue, lethargy, and activity intolerance in recent weeks. His care team has established that his GFR remains at a low, but stable, level. Which of the following assessments is most likely to inform a differential diagnosis?

Blood work for hemoglobin, red blood cells, and hematocrit. Feedback: Anemia is a frequent, and debilitating, consequence of CKD. The anemia may be due to chronic blood loss, hemolysis, bone marrow suppression due to retained uremic factors, and decrease in red cell production due to impaired production of erythropoietin and iron deficiency. Pancreatic function is not typically affected by CKD, and endoscopic examination is less likely to reveal a cause of fatigue. An infectious etiology is possible and would be informed by white cell assessment, but this is less likely than anemia given the client's complaints.

In men experiencing nonrelaxing external sphincter with associated urine retention, the health care worker should assess for which of the following possible causes?

Chronic prostatitis. Feedback: In men, chronic prostatitis contributes to impaired (nonrelaxing) external sphincter with urine retention. The stress response can cause retention of urine as part of the "fight-or-flight" response, unrelated to sphincter dysfunction. Developmental delays are associated with female or male children (not men). If intra-abdominal pressure increases as it does during actions such as coughing, laughing, or sneezing and if this pressure is not equally transmitted to the urethra, then incontinence occurs. PID is primarily a female disorder.

As chronic kidney disease progresses, the second stage (renal insufficiency) is identified by:

Decrease in GFR of 60 to 89 mL/minute/1.73 m2. Feedback: Diminished renal reserve is characteristic of renal insufficiency, when labs remain normal but there is renal insufficiency. Only the second stage, formerly known as renal insufficiency, is characterized by a decrease in GFR of 60 to 89 mL/minute/1.73 m2. The other choices represent stage 3, 4, and 5, respectfully.

A client with a history of chronic pyelonephritis has been admitted several times with recurrent bacterial infection of the urinary tract. The nurse should anticipate educating this client with regard to which common treatment regimen?

Continue taking antibiotics for full 10 to 14 days even if symptoms of infection disappear. Feedback: Chronic pyelonephritis involves a recurrent or persistent bacterial infection superimposed on urinary tract obstruction, urine reflux, or both. Chronic obstructive pyelonephritis can be bilateral, caused by conditions that obstruct bladder outflow; or unilateral, such as occurs with ureteral obstruction. Cranberry juice, forced micturition, and diuretics are not standard treatments for chronic pyelonephritis.

A patient is admitted with nausea, vomiting, diarrhea, and abdominal pain. The patient appears emaciated and complains of feeling weak. The nurse notes a heart rate of 98 beats/min and a blood pressure of 88/54 mm Hg. The nurse reviews the chart and notes an increased serum potassium level and a decreased serum sodium level. The nurse expects the provider to order which medication initially?

Cosyntropin [Cortrosyn] Cosyntropin is used to diagnose adrenal insufficiency, which this patient shows signs of having. Dexamethasone is used to diagnose Cushing's syndrome. Fludrocortisone, a mineralocorticoid, is used with hydrocortisone to treat primary adrenal insufficiency; both of these drugs will be administered after the diagnosis has been confirmed.

A patient is given 1 mg of dexamethasone at 11:00 PM; a plasma cortisol level recorded at 8:00 PM the next day is normal. The nurse knows that this is an indication that the patient has what condition?

Cushing's syndrome The overnight dexamethasone suppression test, which is performed by administering dexamethasone as described, is used to diagnose Cushing's syndrome. In normal individuals, dexamethasone suppresses the release of adrenocorticotropic hormone (ACTH), thereby suppressing the synthesis and release of cortisol, which results in a low cortisol level. In patients with Cushing's syndrome, the cortisol level is normal or only mildly low. This is not an indication of Addison's disease, which is diagnosed by administering cosyntropin. Neither test is useful for diagnosing congenital adrenal hyperplasia or secondary adrenal insufficiency.

Impaired skin integrity and skin manifestations are common in persons with chronic kidney disease. Pale skin and subcutaneous bruising are often present as a result of:

Impaired platelet function. Feedback: Bruising and pale skin are present with chronic kidney disease because platelet function is impaired. Adequate platelets are available, but the function is abnormal. Renal clients do not routinely receive anticoagulant therapy, since they already have bleeding tendencies. Increased vascular volume is associated with renal disease.

The immunosuppressive and anti-inflammatory effects of cortisol cause:

Inhibition of prostaglandin synthesis. Feedback: Large quantities of cortisol are required for an effective anti-inflammatory action. The increased cortisol blocks inflammation at an early stage by decreasing capillary permeability and stabilizing the lysosomal membranes so that inflammatory mediators are not released. Cortisol suppresses the immune response by reducing humoral and cell-mediated immunity. Cortisol also inhibits prostaglandin synthesis, which may account in large part for its anti-inflammatory actions. Cortisol stimulates glucose production by the liver; as glucose production by the liver rises and peripheral glucose use falls, a moderate resistance to insulin and hyperglycemia develop.

A hormone has been synthesized in the rough endoplasmic reticulum of an endocrine cell after which it has moved into the Golgi complex, been packaged in a vesicle, and been released into circulation. From the following list, which hormone is synthesized and released in this manner?

Insulin. Feedback: Insulin is a peptide hormone; as such, its synthesis and release are vesicle mediated. Glucocorticoids (such as cortisol), androgens (such as testosterone), and estrogens are synthesized by non-vesicle-mediated pathways.

A patient with hypothyroidism begins taking PO levothyroxine [Synthroid). The nurse assesses the patient at the beginning of the shift and notes a heart rate of 62 beats/min and a temperature of 97.2°F. The patient is lethargic and difficult to arouse. The nurse will contact the provider to request an order for which drug?

Intravenous levothyroxine d. Methimazole [Tapazole]. Intravenous administration of levothyroxine is used for myxedema coma. This patient is showing signs of severe hypothyroidism, or myxedema. A beta blocker is useful in patients who show signs of hyperthyroidism to minimize cardiac effects. Because the half-life of oral levothyroxine is so long, increasing the PO dose will not provide immediate relief of this patient's symptoms. Methimazole is used to treat hyperthyroidism.

A client with excessive production of growth hormone level will likely exhibit which clinical manifestations? Select all that apply.

Large hands and feet due to increased production of GH, Excess thirst and urination due to decreased glucose uptake, and Difficulty chewing food. Feedback: Growth hormone causes increased release of free fatty acids from adipose tissue, leading to increased concentration of free fatty acids in body fluids. In addition, GH exerts multiple effects on carbohydrate metabolism, including decreased glucose uptake by tissues such as skeletal muscle and adipose tissue, increased glucose production by the liver, and increased insulin secretion. Each of these changes results in GH-induced insulin resistance. Impaired glucose tolerance occurs in as many as 50% to 70% of persons with acromegaly; overt diabetes mellitus subsequently can result. The predominant effect of prolonged growth hormone (GH) excess is to increase glucose levels despite an insulin increase. Persons with classic GH deficiency have normal intelligence, short stature, and obesity with immature facial features. Exceptionally tall children (i.e., genetic tall stature and constitutional tall stature) can be treated with sex hormones—estrogens in girls and testosterone in boys—to effect early epiphyseal closure and stop bone growth.

While teaching a science class, the instructor mentions that both autocrine and paracrine hormonal actions occur without entering the bloodstream. The instructor then asks the students, "What cells do paracrine actions affect?" The student with the correct answer is:

Local. Feedback: Paracrine actions are hormonal interactions with local cells other than those that produce the hormone; autocrine actions are with self-cells (cells from which they were produced). Both autocrine and paracrine hormonal actions affect target cells. Neither paracrine or autocrine actions affect cell storage.

The iatrogenic form of Cushing syndrome is caused by:

Long-term cortisone therapy. Feedback: Three important forms of Cushing syndrome result from excess glucocorticoid production by the body. One is a pituitary form, which results from excessive production of ACTH by a tumor of the pituitary gland, called Cushing disease. The second form is the adrenal form, caused by a benign or malignant adrenal tumor. The third form is ectopic Cushing syndrome, caused by a non pituitary ACTH-secreting tumor, often carcinoma of the lung. Iatrogenic Cushing syndrome results from long-term therapy with one of the potent pharmacological preparations of glucocorticoids.

Which of the following aspects of kidney function is performed by the juxtaglomerular apparatus?

Matching changes in GFR with renal blood flow. Feedback: The juxtaglomerular apparatus is thought to represent a feedback control system that links changes in the glomerular filtration rate (GFR) with renal blood flow.

Reduced glomerular filtration rate (GFR), with a serum creatinine level that remains in the normal range, is associated with aging because elderly persons tend to have reduced:

Muscle mass. Feedback: Serum creatinine level is directly related to muscle metabolism. Because muscle mass is reduced in elderly persons, the creatinine level does not increase as readily with a lower GFR. Drug tolerance and renal perfusion can affect the GFR, but the age-related normal creatinine level can also be present. Calcium intake is unrelated to creatinine levels or GFR.

A chronic kidney disease client who has renal osteodystrophy should be assessed for which of the following complications? Select all that apply.

Muscle weakness, Bone pain, and Stress fractures. Feedback: Both types of renal osteodystrophy are manifested by abnormal absorption and defective bone remodeling. Renal osteodystrophy is typically accompanied by reductions in bone mass, alterations in bone microstructure, bone pain, and skeletal fracture. There are changes in bone turnover, mineralization, and bone volume, accompanied by bone pain and muscle weakness, risk of fractures, and other skeletal complications. Kidney stones and urosepsis are not associated with renal osteodystrophy.

The nurse is caring for a pregnant patient recently diagnosed with hypothyroidism. The patient tells the nurse she does not want to take medications while she is pregnant. What will the nurse explain to this patient?

Neuropsychological deficits in the fetus can occur if the condition is not treated. Maternal hypothyroidism can result in permanent neuropsychological deficits in the child. Hypothyroidism is not a normal effect of pregnancy and is a serious condition that can affect both mother and fetus. The greatest danger to the fetus occurs in the first trimester, because the thyroid does not fully develop until the second trimester. Early identification is essential. Symptoms often are vague. Treatment should begin as soon as possible, or mental retardation and other developmental problems may occur.

A patient who has erectile dysfunction asks a nurse whether sildenafil [Viagra] would be a good medication for him to take. Which aspect of this patient's history would be of most concern?

Occasional use of nitroglycerin. Patients taking nitroglycerin should not take sildenafil. Having BPH is not a contraindication. Mild hypertension requires caution but is not a contraindication. Patients with BPH taking finasteride may take sildenafil.

The most common cause of thyrotoxicosis is Graves disease. When assessing this client, the nurse should put priority on which of the following signs/symptoms?

Ophthalmopathy. Feedback: Graves disease is characterized by a triad of hyperthyroidism, goiter, ophthalmopathy (exophthalmos), or less commonly, dermopathy (pretibial edema due to accumulation of fluid and glycosaminoglycans). Even in persons without exophthalmos (i.e., bulging of the eyeballs seen in ophthalmopathy), there is an abnormal retraction of the eyelids and infrequent blinking such that they appear to be staring. Although the myxedema of hypothyroidism is most obvious in the face and other superficial parts, it also affects many of the body organs. Common to all types of thyrotoxicosis, rather than unique to Graves disease, cholesterol blood levels are decreased; muscle proteins are broken down and used as fuel, which accounts for the muscle fatigue that occurs with all types of hyperthyroidism.

An elderly client who experiences chronic pain takes opioid analgesics on a regular basis, a practice that has resulted in frequent constipation and occasional bowel obstructions. Which of the following problems may directly result from these gastrointestinal disorders?

Overflow urinary incontinence. Feedback: Fecal impaction occurs when a large bolus of stool forms in the rectum, which can push against the urethra causing obstruction that results in overflow incontinence. This does not constitute a risk factor for bladder cancer or neurogenic bladder, and although a urinary tract infection (UTI) may result, this is an indirect consequence of the bowel obstruction.

Which of the following statements best describes the relationship between the hypothalamus and the posterior pituitary in the normal functioning of the endocrine system?

Posterior pituitary hormones are produced in the hypothalamus but released from the pituitary gland. Feedback: The posterior pituitary hormones, ADH and oxytocin, are synthesized in the cell bodies of neurons in the hypothalamus that have axons that travel to the posterior pituitary, where they are released when needed. The two glands do not contribute components that are subsequently combined.

A 43-year-old female has recently been diagnosed with systemic lupus erythematosus (SLE) glomerulonephritis. She has presented to the out-client department to have a renal biopsy. Knowing the usual treatment options, the nurse should anticipate educating the client (who has a positive biopsy result) on which of the following medications being prescribed? Select all that apply.

Prednisone (a corticosteroid) and Captopril ( ACE inhibitor) Feedback: Treatment depends on the extent of glomerular involvement. Oral corticosteroids and angiotensin-converting enzyme (ACE) inhibitors are the mainstays of treatment. Diuretics and antibiotics are not part of the treatment protocol.

Which of the following physiologic processes is performed by the kidneys and contributes to increased blood pressure?

Production and release of renin. Feedback: Renin, an enzyme that is synthesized and stored in the juxtaglomerular cells of the kidney, enzymatically converts angiotensinogen to angiotensin I. Angiotensin I is converted to angiotensin II, a potent vasoconstrictor, in the lungs. Aldosterone is secreted by the adrenal glands and does not convert to angiotensin.

Which of the following physiologic processes is a direct effect of the release of growth hormone by the anterior pituitary?

Production of insulin-like growth factors (IGFs) by the liver. Feedback: GH cannot directly produce bone growth; instead, it acts indirectly by causing the liver to produce IGFs. It affects neither metabolic rate nor the function of the hypothalamic-pituitary-thyroid feedback system.

An athlete has become dehydrated during a long race in hot weather. Which of the following physiologic processes will occur in an attempt to protect the athlete's extracellular fluid volume?

Release of ADH from the posterior pituitary. Feedback: ADH assists in the maintenance of the extracellular fluid volume by controlling the permeability of the medullary collecting tubules. Osmoreceptors in the hypothalamus sense an increase in osmolality of extracellular fluids and stimulate the release of ADH from the posterior pituitary gland. In exerting its effect, ADH, also known as vasopressin, binds to receptors on the basolateral side of the tubular cells. Binding of ADH to the vasopressin receptors causes water channels, known as aquaporin-2 channels, to move into the luminal side of the tubular cell membrane, producing a marked increase in water permeability. The ascending limb of the loop of Henle and the distal convoluted tubules are largely impermeable to water, and arteriole dilation does not directly increase the amount of water reabsorbed from glomerular filtrate.

A client is admitted with worsening heart failure. The client is complaining about having to urinate frequently. The nurse knows that the physiology behind the body's response to decrease vascular volume by increasing urine output is due to:

Release of atrial natriuretic peptide (ANP) from overstretched atria. Feedback: ANP is believed to play an important role in salt and water excretion by the kidney. It is synthesized by muscle cells in the atria of the heart and released when the atria are stretched. Increased levels of this peptide directly inhibit the reabsorption of sodium and water in the renal tubules. ANP also inhibits renin secretion and therefore angiotensin II formation, which in turn reduces reabsorption of sodium. The decrease in sodium reabsorption increases urine output and helps return blood volume to normal. ANP levels, which become elevated when the atria are stretched in HF, help to decrease vascular volume by increasing urine output. Potassium reabsorption is not responsible for water excretion. Aldosterone secretion by the adrenal gland functions in the regulation of sodium and potassium elimination by the principal cells in the distal and collecting tubules.

When comparing the endocrine and nervous system functions, the nurse knows that the endocrine system: Select all that apply.

Releases hormones into the blood that is transported throughout the body, Glands are widely scattered throughout the body, and Takes longer to respond to innervations but has prolonged actions when they arrive Feedback: The endocrine system uses chemical messengers called hormones as a means of controlling the flow of information between the different tissues and organs of the body. It does not act alone, however, but interacts with the nervous system to coordinate and integrate the activity of body cells. Hormones regulate and integrate body functions. Hormones act on specific target cells, but they cause a variety of effects on tissues. Hormones do not transport other substances; hormones are transported and present in body fluids at all times. The endocrine system uses hormones released into the blood and transported throughout the body to influence the activity of body tissues. Tissue and organ responses to endocrine hormones tend to take much longer than the response to neurotransmitters, but once initiated, they tend to be much more prolonged than those induced by the nervous system. The glands of the endocrine system are widely scattered throughout the body.

While taking a client history, which of the following assessments lead the nurse to suspect the client may have polycystic kidney disease? Select all that apply.

Renal colic with flank pain, Bright red blood in urine sample , Elevated blood pressure of 180/94. Feedback: The manifestations of ADPKD include pain from the enlarging cysts that may reach debilitating levels, episodes of gross hematuria from bleeding into a cyst, infected cysts from ascending UTIs, and hypertension resulting from compression of intrarenal blood vessels with activation of the renin-angiotensin mechanism. Renal colic caused by nephrolithiasis, or kidney stones, occurs in about 20% of persons with ADPKD. One type of pain associated with kidney stones is renal colic, described as colicky pain that accompanies stretching of the collecting system or ureter. Nephrotic syndrome is characterized by massive proteinuria. SOB with abnormal respiratory sounds is not usually associated with ADPKD.

An infant has been diagnosed with autosomal recessive polycystic kidney disease (ARPKD). Which of the following treatment goals would be considered the priority in the care of this child?

Respiratory support. Feedback: Aggressive ventilatory support is often necessary for neonates with ARPKD, due to the presence of pulmonary hypoplasia and hypoventilation. Hydration, nutrition, and infection prevention are relevant aspects of care, but respiratory interventions are the priority.

A client has just undergone a diagnostic cardiac angiogram. As part of their ordered labs, the physician has ordered a thyroid panel. The physiological principle behind ordering this lab tests includes which of the following correlations? Hyperthyroidism can cause: Select all that apply.

Rise in oxygen consumption and Increase in cardiac output. Feedback: Cardiovascular and respiratory functions are strongly affected by thyroid function. With an increase in metabolism, there is a rise in oxygen consumption and production of metabolic end products, with an accompanying increase in vasodilation. Blood volume, cardiac output, and ventilation all are increased as a means of maintaining blood flow and oxygen delivery to body tissues. Heart rate and cardiac contractility are enhanced as a means of maintaining the needed cardiac output, whereas there is little change in blood pressure because the increase in vasodilation tends to offset the increase in cardiac output.

An older adult patient is diagnosed with hypothyroidism. The initial free T4 level is 0.5 mg/dL, and the TSH level is 8 microunits/mL. The prescriber orders levothyroxine [Levothroid] 100 mcg/day PO. What will the nurse do?

Suggest that the provider lower the dose. In older adult patients, initial dosing of levothyroxine should start low and be increased gradually. A typical starting dose for an elderly patient is 25 to 50 mcg/day. It is not correct to administer the medication without questioning the provider. Unless the patient has signs of myxedema, there is no need to give the medication IV. Desiccated thyroid is no longer used except in patients who have been taking it long term.

Which of the following factors is likely to result in decreased renal blood flow?

Sympathetic nervous system stimulation. Feedback: Sympathetic nervous system (SNS) stimulation results in decreased renal blood flow by vasoconstriction. Dopamine, nitric oxide, and prostaglandins are all vasodilators.

A client has developed the facial appearance that is characteristic of myxedema, along with an enlarged tongue, bradycardia, and voice changes. Which of the following treatment modalities is most likely to benefit this client?

Synthetic preparations of T3 or T4. Feedback: Myxedema and the client's other signs are associated with hypothyroidism, which necessitates thyroid hormone replacement. B-Adrenergic blocking drugs and antithyroid drugs are indicated in the treatment of hyperthyroidism, whereas treatments relevant to adrenal cortical function are not relevant to hypothyroidism.

Thyroid and steroid hormones, which exert their effect on target cells by way of nuclear receptors, have which of the following characteristics?

The ability to cross the cell membrane of target cells. Feedback: Hormones that utilize nuclear receptors enter the target cell (i.e., cross the cell membrane) and bind to receptors in the cell nucleus that are gene regulatory proteins. These hormones do not selectively utilize second messengers, and they do not interact with surface receptors. They are not both lipid and water soluble.

Which of the following events would suggest that an individual's physiologic response to an obstruction has progressed beyond the compensatory stage and is now in the decompensatory stage?

The detrusor loses its power of contraction. Feedback: The compensatory stage of the response to a bladder obstruction involves hypertrophy of the bladder wall, urgency, and difficulty suppressing the urge to urinate. If these compensatory measures are ineffective, decompensation occurs. The per muscle contraction becomes too short to expel the urine completely, and residual urine remains in the bladder. At this point, the symptoms of obstruction-frequency of urination, hesitancy, need to strain to initiate urination, a weak and small stream, and termination of the stream before the bladder is completely emptied—become pronounced.

A patient in her twenties with Graves' disease who takes methimazole [Tapazole) tells a nurse that she is trying to conceive and asks about disease management during pregnancy. What will the nurse tell her?

The patient should discuss changing to propylthiouracil from now until her second trimester with her provider. Methimazole is not safe during the first trimester of pregnancy, because it is associated with neonatal hypothyroidism, goiter, and cretinism; however, it is safe in the second and third trimesters. Propylthiouracil is recommended for pregnant patients only in the first trimester and during lactation only if a thioamide is absolutely necessary. Iodine-131 is used in women older than 30 years who have not responded to medication therapy and is contraindicated during pregnancy.

When explaining factors that influence the number of receptors present on target cells, the instructor will likely mention: Select all that apply.

The role antibodies may have on receptor proteins. A decreased hormone level may produce increased receptor numbers. A sustained excess hormone level brings about a decrease in receptor numbers. Feedback: Target cell response varies with the number and affinity of the relevant receptors. The number of hormone receptors on a cell may be altered for any of several reasons. Antibodies may destroy or block the receptor proteins. Increased or decreased hormone levels often induce changes in the activity of the genes that regulate receptor synthesis. For example, decreased hormone levels often produce an increase in receptor numbers by means of a process called up-regulation; this increases the sensitivity of the body to existing hormone levels. Likewise, sustained levels of excess hormone often bring about a decrease in receptor numbers by down-regulation, producing a decrease in hormone sensitivity.

Which of the following statements about immunoradiometric assay (IRMA) testing for is measuring plasma hormone levels the most accurate?

These tests are very specific since they utilize two antibodies instead of one. Feedback: Immunoradiometric assay (IRMA) testing is very specific since they utilize two antibodies instead of one. These two antibodies are directed against two different parts of the molecule, and therefore IRMA assays are more specific. Hormones circulating in the plasma were first detected by bioassay test, which used an intact animal or a portion of tissue from an animal to calculate specificity and sensitivity. ELISA testing procedure utilizes antibody-coated plates to produce colored reactions. The IRMA is a blood test, not a urine test.

Which of the following medications may be responsible for a client developing increased uric acid levels by decreasing ECF volume?

Thiazide diuretics. Feedback: Because of its effect on uric acid secretion, aspirin is not recommended for treatment of gouty arthritis. Thiazide and loop diuretics also can cause hyperuricemia and gouty arthritis, presumably through a decrease in ECF volume and enhanced uric acid reabsorption.

A clinic patient who has cirrhosis of the liver develops hypervolemic hypernatremia. Which medication will the nurse expect the provider to order?

Tolvaptan [Samsca]. Tolvaptan is an oral vasopressin antagonist indicated for reducing hypernatremia in patients with euvolemic or hypervolemic hypernatremia. Conivaptan has the same indications, but it is administered IV for hospitalized patients. Desmopressin and vasopressin are antidiuretic hormone replacement medications that will make the condition worse.

Clients with CKD are at risk for demineralization of their bones since they are no longer able to:

Transform vitamin D to its active form. Feedback: The kidneys aid in calcium metabolism by activating vitamin D, after it is chemically converted by the liver. Bicarbonate buffering is unrelated to activation of vitamin D. Bone marrow is stimulated by the synthesis of erythropoietin to form red blood cells, which is unrelated to calcium levels.

While discussing the elimination of hormones from the body to prevent overaccumulation, which of the following hormones are eliminated in bile? Select all that apply.

Unbound adrenal hormones, Gonadal steroid hormones, and Thyroid hormones. Feedback: Steroid hormones are bound to protein carriers for transport and are inactive in the bound state. Unbound adrenal and gonadal steroid hormones are conjugated in the liver, which renders them inactive, and then excreted in the bile or urine. Thyroid hormones also are transported by carrier molecules. The free hormone is rendered inactive by the removal of amino acids in the tissues, and the hormone is conjugated in the liver and eliminated in the bile. The catecholamine production is measured by some of their metabolites. In general, peptide hormones also have a short life span in the circulation. Their major mechanism of degradation is through binding to cell surface receptors, with subsequent uptake and degradation by peptide-splitting enzymes in the cell membrane or inside the cell.

A 20-year-old female patient has suprapubic discomfort, pyuria, dysuria, and bacteriuria greater than 100,000/mL of urine. Which are the most likely diagnosis and treatment?

Uncomplicated lower urinary tract infection treatable with short-course therapy These are symptoms of uncomplicated cystitis, which is a lower urinary tract infection that can be treated with a short course of antibiotics. Short-course therapy is more effective than single dose therapy and is preferred. A complicated lower urinary tract infection would be associated with some predisposing factor, such as renal calculi, an obstruction to the flow of urine, or an indwelling catheter. Upper urinary tract infections often include severe flank pain, fever, and chills.

A 40-year-old female with the diagnosis of multiple sclerosis has been experiencing severe bladder spasms along with less bladder volume. This is likely due to: Select all that apply.

Uninhibited bladder and Neurogenic detrusor overactivity. Feedback: Neurogenic detrusor overactivity, or spastic bladder, is usually characterized by reflex bladder spasms and a decrease in bladder volume. The most common causes of neurogenic detrusor overactivity are spinal cord lesions such as spinal cord injury; vascular lesions, tumors, or herniated intervertebral disk; and multiple sclerosis. Bladder atonia is caused by spinal cord injury. Autonomic hyperreflexia is due to spinal cord injuries at the cervical level. Uninhibited bladder can develop after a stroke or during the early stages of multiple sclerosis.

An obese, male client with a history of gout and a sedentary lifestyle has been advised by his primary care provider to avoid organ meats, certain fish, and other foods that are high in purines. The care provider is demonstrating an awareness of the client's susceptibility to which of the following types of kidney stones?

Uric acid stones. Feedback: Uric acid stones develop in conditions of gout and when high concentrations of uric acid in the urine. Unlike radiopaque calcium stones, uric acid stones are not visible on X-ray films. According to Table 25-2, these stones develop in clients who eat a high-purine diet like Atkins.

The body compensates for obstructed urine outflow up to a certain point. Which of the following signs/symptoms lead the nurse to suspect decompensatory changes are occurring? Select all that apply.

Urinary frequency noted, High residual volume up to 1000 mL, and Must strain to initiate the stream of urine. Feedback: Compensatory changes to chronic obstruction include increased urge to urinate (urinary frequency). When compensatory mechanism no longer is effective, signs of decompensation begin to appear. The period of detrusor muscle contraction becomes too short to expel the urine completely, and residual urine remains in the bladder. A PVR value of less than 50 mL is considered adequate bladder emptying, and more than 200 mL indicates inadequate bladder emptying. At this point, the symptoms of obstruction-frequency of urination (during day and night), hesitancy, need to strain to initiate urination, a weak and small stream, and termination of the stream before the bladder is completely emptied—become pronounced. Bladder spasms are a symptom of outlet obstruction and do not increase urine output/decrease obstruction.

The most damaging effects of urinary obstruction are the result unrelieved obstruction of urine outflow and:

Urinary stasis. Feedback: The most damaging effects of urinary obstruction are stasis of urine, which predisposes to infection and stone formation, and unrelieved obstruction of urine outflow. Most commonly, the person has pain, signs, and symptoms of urinary tract infection (UTI) and manifestations of renal dysfunction, such as an impaired ability to concentrate urine. Progressive atrophy of the kidney is caused by obstruction of the outflow of urine. Hypertension is an occasional complication of urinary tract obstruction, since urine flow is obstructed rather than renal blood flow.

A client has provided a routine urine sample during a scheduled visit to his primary care provider. Which of the following results is an expected finding in a healthy individual?

Urine specific gravity of 1.020. Feedback: Normal urine specific gravity ranges from 1.010 to 1.025. Glucose and casts are normally absent, and albumin is normally present in only scant amounts.

Many factors contribute to the incontinence that is common among the elderly. A major factor is increased:

Use of multiple medications. Feedback: Use of multiple medications for other health problems can affect bladder function, especially diuretics. Drugs such as hypnotics, tranquilizers, and sedatives can interfere with the conscious inhibition of voiding, leading to urge incontinence. Detrusor muscle function and urethral closing pressure are decreased in the elderly, causing incontinence. Decreased fluid and water intake causes problems of bowel impaction and urinary tract infection.

In major athletic competition, athletes are required to submit to liquid chromatography testing looking for:

Use of performance-enhancing agents to increase the chances of winning. Feedback: For some steroid or peptide hormones, mass spectrometry is becoming increasingly useful and can be combined with other analytical techniques, such as liquid chromatography. These approaches provide definitive identification of the relevant hormone or compound according to its chemical or physical characteristics (e.g., unequivocal detection of performance-enhancing agents in sports).

A patient arrives in the emergency department with a heart rate of 128 beats/min and a temperature of 105°F. The patient's skin feels hot and moist. The free T4 level is 4 ng/dL, the free T3 level is 685 pg/dL, and the TSH level is 0.1 microunits/mL. The nurse caring for this patient will expect to administer:

propylthiouracil (PTU). Propylthiouracil is used for patients experiencing thyroid storm, and this patient is showing signs of that condition. Levothyroxine is given IV for hypothyroidism. 1311 is used in patients over age 30 who have not responded to other therapies. Methimazole is used long term to treat hyperthyroidism, but PTU is more useful for emergency treatment.

An older male patient comes to the clinic with complaints of chills, malaise, myalgia, localized pain, dysuria, nocturia, and urinary retention. The nurse would most likely suspect that the patient has:

prostatitis. The nurse should suspect prostatitis, which is manifested by high fever, chills, malaise, myalgia, and localized pain and may also be manifested by dysuria, nocturia, and urinary urgency, frequency, and retention. Clinical manifestations of acute cystitis include dysuria, urinary urgency and frequency, suprapubic discomfort, pyuria, and bacteriuria. Urinary tract infections are very general and are classified by their location. Pyelonephritis is characterized by fever, chills, severe flank pain, dysuria, and urinary frequency and urgency, as well as by pyuria and bacteriuria.

A patient presents to the emergency department with complaints of chills, severe flank pain, dysuria, and urinary frequency. The patient has a temperature of 102.9°F, a pulse of 92 beats/min, respirations of 24 breaths per minute, and a blood pressure of 119/58 mm Hg. The nurse would be correct to suspect that the patient shows signs and symptoms of:

pyelonephritis. The nurse should suspect pyelonephritis. Pyelonephritis is characterized by fever, chills, severe flank pain, dysuria, urinary urgency and frequency, and pyuria and bacteriuria. Clinical manifestations of acute cystitis include dysuria, urinary urgency and frequency, suprapubic discomfort, pyuria, and bacteriuria. Urinary tract infections (UTIS) are very general and are classified by their location. These symptoms are specific to pyelonephritis. Prostatitis is manifested by high fever, chills, malaise, myalgia, localized pain, and various UTI symptoms but not by severe flank pain.

A patient with antidiuretic hormone deficiency is receiving desmopressin (DDAVP). The nurse will teach this patient to:

reduce fluid intake. Patients taking desmopressin should be taught to reduce fluid intake to avoid fluid overload, because this drug prevents continued fluid loss. Desmopressin metabolism is not affected by grapefruit juice. Increasing the sodium intake would only cause increased fluid retention. Desmopressin is a weak pressor agent and does not adversely affect hemodynamics, so monitoring blood pressure is not necessary.

A patient is admitted to the hospital and will begin taking levothyroxine [Synthroid). The nurse learns that the patient also takes warfarin [Coumadin). The nurse will notify the provider to discuss the dose.

reducing warfarin Levothyroxine accelerates the degradation of vitamin K dependent clotting factors, which enhances the effects of warfarin. Patients taking warfarin who start taking levothyroxine may need to have their warfarin dose reduced. It is not correct to increase or decrease the levothyroxine dose or to increase the warfarin dose.

A 68-year-old male patient receives a prescription for 25-mg tablets of sildenafil [Viagra] for erectile dysfunction. When he asks the nurse how to take the medication, the nurse will tell him to:

start with one tablet about 1 hour before anticipated sexual activity. Patients older than 65 years should start with a low dose of 25 mg and may take the drug 1 hour before anticipated sexual activity. Sildenafil is taken when needed and not on a routine basis. Dosing at 25 to 100 mg per dose 30 minutes to 4 hours before sexual activity is the standard recommendation for men younger than 65 years. Two tablets is a high dose; consumption of a high-fat meal would interfere with absorption of sildenafil.

A nurse is caring for a patient who has myasthenia gravis. The prescriber has ordered Pyridostigmine [Mestinon, Regonol]. An important initial nursing action before administration of the medication includes assessing:

swallowing ability. Many patients hospitalized for myasthenia gravis do not have the muscle strength to swallow well and need a parenteral form of the medication; therefore, assessing the patient's ability to swallow is an important initial safety measure. Evaluating the patient's ability to raise the eyelids, level of fatigue, and skeletal muscle strength are important assessments before drug administration and during drug treatment, because they indicate the effectiveness of the drug and help determine subsequent doses.


Related study sets

Chapter 27. Measuring Domestic Output and National Income

View Set

Module11 Security in Network Design

View Set

Earthquakes and Earth's Interior

View Set

Life and Health Simulation Exam Missed Questions

View Set

CHFI - Chapter 6 (Operating System Forensics)

View Set

A&P Chapter 14: Brain and Cranial Nerves

View Set

Prep U's - Chapter 1 - Professional Nursing Practice

View Set

Chapter 23: perioperative nursing

View Set